24 декабря Архивач восстановлен после серьёзной аварии. К сожалению, значительная часть сохранённых изображений и видео была потеряна. Подробности случившегося. Мы призываем всех неравнодушных помочь нам с восстановлением утраченного контента!

Добротред

 Аноним 24/01/17 Втр 20:55:18 #1 №9338 
1453509477166846934.jpg
08fa53786648baXL.jpg
86b32a0a4ccc58a47222cc49349445c528ce9a3528468cd9a162bbacebf[...].jpg
whenimlearningmathbydiamimi-d6sad14.jpg
В связи с тем, что оп начинайкотреда продолжает слоупочить, я взял на себя смелость создать новый, альтернативный тред для начинающих.

Здесь мы будем не спеша и без фанатизма изучать математику, задавать ответы и получать вопросы, а также культурно общаться на смежные темы. ВНИМАНИЕ! Тред — модерируемый! Я собственноручно и прилежно буду репортить любой пост, являющийся нерелейтедом или содержащий мат и оскорбления, и очень прошу модератора неизменно такие посты удалять. Пожалуйста, воздержитесь от грубостей в этом треде! Если вам такой стиль общения не по душе - скройте этот тред.

Остальным же - добро пожаловать в добротред. И помните: главное — хуявное
Аноним 24/01/17 Втр 22:38:01 #2 №9342 
Всем добра и позитива итт
Аноним 25/01/17 Срд 01:59:25 #3 №9351 
>>9338 (OP)
Привет! У меня высшего мат.образования, поэтому прошу гнилые помидоры не кидать - пишу в треде для начинающих.
У меня такой вопрос. Решил я построить модель для школьной евклидовой геометрии в теории множеств (ZFC). Определяю множество действительных чисел R, строю пространство R^3, определяю множество прямых как множество множеств точек, удовлетворяющих условиям. Но пытаясь определить углы, понимаю, что без тригонометрических функций не обойтись. Это так? Чтобы построить евклидову геометрию в ZFC, нужно сначала построить тригонометрические функции (то есть не тригонометрия логически опирается на геометрию, а геометрия логически опирается на тригонометрию)?
Аноним 25/01/17 Срд 03:24:24 #4 №9353 
>>9351
В R^3, как минимум, нужно ввести скалярное произведение, без него ты углы не определишь никак.
Аноним 25/01/17 Срд 07:23:34 #5 №9360 
>>9351
Мне тоже кажется, что вместо прямых тебе нужно ввести векторное пространство прямые тогда будут парой вектора и точки, а в нем - скалярное произведение.

Насчет того, что на что опирается - это вопрос философский. Ни что ни на что не опирается, оно все просто существует. На что тебе проще и удобнее опирать, на то оно и будет опираться, вот.
sagesage sage 25/01/17 Срд 10:15:31 #6 №9364 DELETED
Пиздец. Пиздец. Натуральный /b.
Аноним 25/01/17 Срд 10:24:58 #7 №9365 
>>9351
можно ввести метрику и несколько угловых аксиом
Аноним 25/01/17 Срд 11:28:18 #8 №9366 
>>9360

Двачую. Такой подход вроде как называется построение геометрии по Вейлю. Сначала определяй векторное пространство, затем скалярное произведение на нем, затем угол между векторами.

Можно также избрать иной путь -- определять геометрию по Клейну: множество с действующей на нем группой преобразований. В случае евклидовой геометрией в R^3, это будут тройки вещественных чисел, на котором действует евклидова, т.е. аффинная ортогональная группа.

Школьный, он же аксиоматический, он же классический евклидов подход значительно менее удобен в работе. Отсюда и всякая ебанина с неполнотой и попытками вывести один постулат из других.
Аноним 28/01/17 Суб 17:07:02 #9 №9585 
>>9366
А ведь можно прямо с углов и начать? Я, правда, не придумал, как одними углами ввести расстояние, не добавляя его явно отдельным числом - тогда все к обычным полярным координатам сводится.

Или вот еще. Можно вместо точек в качестве исходного понятия взять "пятна", то есть множества точек, а сами точки уже ввести как пределы. Вроде физично получается, можно провести аналогии с реальным миром.

Вот например, возвращаясь к углам: можно в качестве "пятен" взять сектора, ну то есть разбиения пространства (ну, пусть пока будет плоскости). Тогда оно у нас будет определяться двумя числами: количеством секторов и номером сектора, который нас нужен. Например, (2;0) - это значит разбить плоскость на две полуплоскости и взять левую. Тут сразу надо показать, что от способа нумерации ничего не зависит. Прямая (ну, или угол) тогда у нас получится, если мы первое число (количество сеекторов-разбиений) устремим к бесконечности. Правда для того, чтобы получить точку, нужно еще какое-то другое разбиение придумать.

Вот, а ведь еще можно показать, что разные алгоритмы разбиений дают тот же результат. Наверное, получатся несколько классов этих алгоритмов, которые дают разную точность. Ну, то есть наверняка ведь есть такие точки, до которых можно добраться, используя один метод разбиения, но которые не получится достать с каким-нибудь другим разбиением.

Только мне что-то никак не приходит в голову, как нам из прямой получить точку. Может нужно взять два разных разбиения, выбрать в каждом из них по сектору и посмотреть, где они (эти сектора, точнее - их "радиусы") пересекаются? Тогда у нас как раз получится и направление, и расстояние на этом направлении. Но вроде это как-то муторно.
sageАноним 06/02/17 Пнд 20:04:19 #10 №10497 DELETED
>>9338 (OP)
Моча, прекращай шалить
Зачем удалил? мог просто закрыть но нахуя?
Аноним 06/02/17 Пнд 20:06:45 #11 №10499 
Тут кто-то лекции Босса просил.
Вот они.
http://rgho.st/private/8ytVrBpz9/bd7c27b7cba52c55ce62bbbe6f86dac2
Аноним 06/02/17 Пнд 20:38:46 #12 №10503 DELETED
>>9338 (OP)
Куда девался предыдущий тред?
Аноним 06/02/17 Пнд 20:49:51 #13 №10504 DELETED
>>10497
>>10503
Небольшие технические неполадки. В ближайшее время либо восстановлю, либо пересоздам. Приносим извинения за доставленные неудобства.
Аноним 06/02/17 Пнд 21:12:03 #14 №10507 
Дано: абсолютные координаты охулиарда точек в трехмерном пространстве.
Нужно: имея только их творить с точками всяческие изъебства, например хуячить оче сложные фигуры и прочее.
Чому я сюда пишу: первый курс был давно, подскажите, что читать.

Спасибо.
Аноним 06/02/17 Пнд 21:47:14 #15 №10513 
>>10507
Демки хочешь писать? Если совсем ничего не помнишь, начни с любого учебника по линейной алгебре (можешь Письменного в качестве справочника подрукой держать), а потом переходи на книжки по тридэ-графики (red book, например).
Аноним 06/02/17 Пнд 21:51:01 #16 №10516 DELETED
>>9338 (OP)
1-ую пикчу использовали абдуловеры, кстати.
Аноним 10/02/17 Птн 01:32:17 #17 №10729 
30-1.jpg
Почему |x| - б >=|а|/2?
Аноним 10/02/17 Птн 02:29:09 #18 №10731 
>>10729
такая дельта
Аноним 10/02/17 Птн 02:58:19 #19 №10732 DELETED
Добротред? Анончики, выручайте, тут пиздец.
>>10727
Аноним 24/02/17 Птн 08:38:34 #20 №11872 
.JPG
проверьте пожалуйста
Аноним 27/02/17 Пнд 16:25:05 #21 №12157 
ананасы, подскажите годный сайт для подготовки к огэ по матеше, плес
Аноним 27/02/17 Пнд 16:32:50 #22 №12158 
>>12157
Матхпрофи, яндекс-егэ.
Аноним 27/02/17 Пнд 21:33:32 #23 №12175 
>>12158
благодарю
Аноним 28/02/17 Втр 20:46:11 #24 №12210 
.dib
Какого хуя?
Аноним 28/02/17 Втр 20:48:40 #25 №12212 
>>11872
Извини, но никто это читать не будет. Напиши словами, что тебе надо и что ты делаешь
Аноним 01/03/17 Срд 12:34:40 #26 №12237 
>>12212
Почему? Хочу доказать что два определения эквивалентны, которые на тетради сверху написаны.
Аноним 04/03/17 Суб 18:01:18 #27 №12529 
Помогите решить
найти значение k, при котором угол между векторами a(1,k),b(3,3) равен 45

ab=|a||b|cos45, но что-то выходит квадратное уравнение с D<0;
Аноним 06/03/17 Пнд 07:57:42 #28 №12675 
>>12529
плохо считаешь значит, всё там нормально получается.
Аноним 06/03/17 Пнд 10:30:41 #29 №12677 
xyz.png
>>12210
кто-то врот
Аноним 06/03/17 Пнд 10:42:55 #30 №12678 
>>12529
находишь скалярное произведение 3+3k через координаты. Затем через модули 3*-/(1+k^2). Тройку нахуй и получаешь 1+k^2=(1+k)^2
и находишь, что k=0
Аноним 06/03/17 Пнд 10:58:39 #31 №12681 
>>11872
красивый почерк. Ты тня или анимедаун?
Аноним 07/03/17 Втр 18:38:02 #32 №12744 
>>12681
Серьезно? Спасибо ^_____^
Аноним 07/03/17 Втр 19:18:13 #33 №12747 
Ананасы, сап. Нужна помощь в решении простого интегральчика. Могу ли здесь обратиться за помощью? Я просто ньюфаг на вашей доске.
Аноним 07/03/17 Втр 23:02:29 #34 №12756 DELETED
>>12747
Обатиться-то ты, конечно, можешь...
Аноним 08/03/17 Срд 08:53:10 #35 №12760 
>>12747
maple и/или mathematica? не?
Аноним 08/03/17 Срд 09:46:10 #36 №12761 
А можно для сюръективной функции построить right inverse, не используя аксиому выбора?
Аноним 08/03/17 Срд 10:22:19 #37 №12762 
>>12761
В общем случае - нет. Рассмотрим семейство непустых непересекающихся множеств M = {Mi}. Его объединение обозначим как N. Рассмотрим отображение N->M, которое каждому элементу x сопоставляет множество Mi, из которого взят этот x. Сечение этой сюръекции будет функцией выбора на N.
Аноним 08/03/17 Срд 10:22:36 #38 №12763 
>>12762
*функцией выбора на M
Аноним 08/03/17 Срд 11:42:05 #39 №12764 
>>12762
То есть эквивалентны?
Аноним 08/03/17 Срд 11:49:39 #40 №12765 DELETED
>>12760
БЕСПЛАТНО блядь. Неужели надо пояснять отдельно на русскоязычной доске?
Аноним 08/03/17 Срд 13:45:40 #41 №12767 DELETED
>>12765
За бесплатно здесь не получится
Аноним 08/03/17 Срд 14:34:14 #42 №12771 
>>12764
Да. Утверждение, что каждая сюръекция имеет сечение, - одна из форм аксиомы выбора.
Аноним 09/03/17 Чтв 21:39:06 #43 №12801 
Снимок.JPG
Выручайте, братья. Нужно вычислить предел. Ответ 1. Не выручите меня мне край.
sageАноним 10/03/17 Птн 16:52:19 #44 №12821 DELETED
>>12801
Пошел нахуй, хуйло ленивое. Не брат ты тут никому, гнида черножопая. Иди хуй соси со своей домашкой.
Аноним 11/03/17 Суб 03:21:07 #45 №12838 
>>12801
Домножим и поделим на выражение, сопряженное числителю. В числителе станет разность квадратов, иксы сократятся.
Аноним 11/03/17 Суб 22:37:21 #46 №12863 
.png
ЧТО ДЕЛАТЬ БЛЯДЬ?! С первым понятно, а дальше как нахуй?
Аноним 12/03/17 Вск 03:26:05 #47 №12881 
>>12863
Измерь наклон касательной, а вообще ты тупое быдло.
Аноним 12/03/17 Вск 10:04:00 #48 №12883 
>>12881
Это понятно блядь, но как? Нижний тангенс угла 2.5. Ну и как теперь верхний узнать блджад?
Аноним 13/03/17 Пнд 03:21:54 #49 №12926 
>>12883
Наклон прямой линии это изменение в y/изменение в x, если изменение в x=1 то изменение в y=-2.5, так понятно?
Аноним 13/03/17 Пнд 14:32:11 #50 №12937 
>>12863
перевести на русский, заебала ваша мова
Аноним 13/03/17 Пнд 17:41:01 #51 №12955 
>>12926
Но на первом х = 1 соответствует у = 5, а на втором - у = - 5.
Аноним 13/03/17 Пнд 17:42:33 #52 №12957 
>>12937
На рисунках график f(х) и касательная в точке -2. Поставить в соответствие значение f'(-2). То есть как я понял это подставить в уравнение касательной -2. Но на первом рисунке это будет просто 5, хотя ответ 0.
Аноним 13/03/17 Пнд 19:31:58 #53 №12961 
>>12957
вроде бы спрашивают чему равна производная в точке x = -2.

посмотри что такое производная в твоей книге.

f - ф-я. f` ее производная.

уравнение прямой: y(x) = kx + b
по графикам прямых найти уравнение:
y(x) = kx + b
k = f`(-2)

в первом графике:
y(x) = 5 => k = 0

в втором:
y(0) = -5
-5 = k 0 + b => b = -5

y(-2) = 0
0 = k (-2) - 5 => k = 5/-2
Аноним 14/03/17 Втр 15:50:05 #54 №12982 
>>12961
> f - ф-я. f` ее производная.
>
> уравнение прямой: y(x) = kx + b
> по графикам прямых найти уравнение:
> y(x) = kx + b
> k = f`(-2)
Но f' это И ЕСТЬ касательная к f, по определению. А ты находишь уже производную от неё, то есть f''.
Аноним 14/03/17 Втр 19:45:50 #55 №12988 
>>12982
на твоей картинке, график касательной нарисован зеленым. я и нашел уравнение этой зеленой линии. k, из этого уравнения и есть f`(-2).
производная, f` - не уравнение касательной.
f` в точке, это уголовой коеф. уравнения касательной.
Аноним 20/03/17 Пнд 02:06:24 #56 №13314 
Интересует социальный вопрос о поступлении на математика (очно).
Каков в основном возрастной контингент на факультете и каковы взаимоотношения между студентами и студентами и преподавателями?
Например, если мне 24 года( не медалист, не олимпиадник), не будет ли это слишком поздно для на математика?
В смысле, не будет ли насмешек и предвзятого отношения в мою сторону?
Аноним 20/03/17 Пнд 02:06:49 #57 №13315 
Где сейчас находит практическое применение алгебра и теория чисел?
Аноним 20/03/17 Пнд 04:06:34 #58 №13318 
типа-книги.jpg
>>13314
Зависит от того, что ты уже знаешь и куда поступаешь. Вообще, бывают математики-первокурсники даже 30lvl, но они уже, как минимум, почитывали пикрелейтед.

>>13315
Везде. Но - только очень простая. Теоретическая математика ушла гораздо дальше, чем нужно практикам.
Аноним 20/03/17 Пнд 16:47:17 #59 №13331 
>>13318
>пикрелейтед
Спасибо! Ещё чуть больше года на подготовку есть, может что-нибудь из "пикрелейтед" успею изучить
Найти константу Аноним 20/03/17 Пнд 20:32:10 #60 №13340 
-q8fBX2yp9Q.jpg
Матемач помоги найти константу, уже 100 лет как забыл всю эту помойку, а тут срочно нужно решить
Аноним 21/03/17 Втр 01:41:03 #61 №13364 
>>13331
Тривиум вербяшки-калоедина еще ёбни
http://gen.lib.rus.ec/book/index.php?md5=FA81A6949A2210F2D7FF89B44EDBF9E0
http://gen.lib.rus.ec/book/index.php?md5=E9FABE7DEFCA16677237BF618028C684
На пикриле действительно все годнота, но некоторое слишком старое и не нужно начинающему,
ну и прочитать 3 книги по матану это не лучшее, что ты можешь сделать за год подготовки
Из матана прочитай лучше 1 том Зорича (это вообще калькулус, а не analysis) и лекции Львовского
А потом ебашь алгебру
21/03/17 Втр 01:51:14 #62 №13365 
>>13364
Отвратительной совет. Особенно с алгеброй. С чего ты взял, что он хочет ей заниматься? И 2-й том Зорича не стоит пропускать.
Аноним 21/03/17 Втр 10:38:58 #63 №13374 
Если не сложно, может кто-нибудь ещё подсказать пособие по всему школьному курсу математики, чтобы знания освежить
Аноним 21/03/17 Втр 13:13:57 #64 №13377 
>>13374
Написанное на каком уровне? Может, проще школьные учебники перечитать?
Аноним 22/03/17 Срд 22:07:39 #65 №13453 
1490209437.jpg
Нужно указать наклонную А1О на плоскость АВС. Не выручите меня, братья мне край.
Аноним 22/03/17 Срд 23:29:06 #66 №13458 
>>13374
И. М. Гельфанд, А. Шень: “Алгебра”
Аноним 22/03/17 Срд 23:30:19 #67 №13459 
>>13453
И в чем проблема-то?
Аноним 22/03/17 Срд 23:41:09 #68 №13460 
>>13459
Я не знаю как ее указать.
Аноним 23/03/17 Чтв 00:08:48 #69 №13463 
>>13460
Так что именно непонятно-то? Что такое наклонная? Что такое плоскость? Или что? Тебя как партизана допытывать приходится, ей-богу.
Аноним 23/03/17 Чтв 01:26:23 #70 №13474 
Подскажите плес, чтобы заниматься исследованием операций, на какую специальность надо поступать, на какой кафедре учиться, на какую магистратуру и аспирантуру идти?
Аноним 25/03/17 Суб 22:02:07 #71 №13597 
>>13377
Нет, в школьных учебниках все очень сильно размазано.
Надо, чтобы годно излагалась теория и были задачи профильного уровня.
Аноним 25/03/17 Суб 22:03:31 #72 №13598 
>>13458
>И. М. Гельфанд, А. Шень: “Алгебра”
Так там геометрии нет
Аноним 25/03/17 Суб 22:05:31 #73 №13599 
>>13598
не нужна))
Аноним 25/03/17 Суб 22:08:41 #74 №13600 
>>13599
А как в нее вкатываться?
Аноним 25/03/17 Суб 22:56:30 #75 №13601 
>>13600
>А. Шень: “Геометрия в задачах”
Аноним 28/03/17 Втр 02:01:59 #76 №13725 
Здравствуйте

посоветуйте что интересного можно почить про операционное исчисление, оригиналы?

в универе данную тему не дали..а самому въезжать сложна
Аноним 28/03/17 Втр 09:14:27 #77 №13731 
>>13725
Маслов Операторные методы
Аноним 28/03/17 Втр 09:50:12 #78 №13732 
>>13731
Спасибо.
Аноним 31/03/17 Птн 07:19:32 #79 №13930 DELETED
Отлично гей-тред даже здесь есть.

Посоветуйте, плиз, "Теория Категорий для чайников".
Аноним 31/03/17 Птн 08:36:04 #80 №13936 DELETED
кто в жопу даёт?
Аноним 04/04/17 Втр 09:12:16 #81 №14143 
14655350435740.png
Господа, подскажите пожалуйста хорошие учебнички по физике за старшие классы-первые курсы. Хочу подвспоминать задачки вроде "насколько поднимется давление в сосуде (размеры) с водой (уровень) если его нагреть с n градусов до m градусов" и всё в таком духе.
Обнял.

мимо человечек
sage[mailto:sage] Аноним 04/04/17 Втр 11:13:11 #82 №14144 
>>14143
В /sci
Школа - Ландсберг, Младш курсы - Фейнман/Матвеев/Беркли
Аноним 05/04/17 Срд 12:23:54 #83 №14229 
PentagonTilings15.svg.png
Зачем в геометрии изучают мозаики и паркеты, что это дает для науки, для практического применения?
Аноним 05/04/17 Срд 13:19:37 #84 №14230 
>>14229
Пол можно выложить
Аноним 05/04/17 Срд 21:18:36 #85 №14260 
Посоветуйте, пожалуйста, элементарный учебник-введение по вариационному исчислению
Аноним 05/04/17 Срд 23:08:49 #86 №14263 
>>14229
Так математика не занимается "практическим применением". Математики изучают всякие прикольные штуки, а потом уже из того, что они наизучали, прикладники берут себе ригодившиеся штуки.
Аноним 06/04/17 Чтв 15:43:18 #87 №14307 
>>14230
Облегчить жизнь Равшану.
Аноним 06/04/17 Чтв 20:00:27 #88 №14320 
blob
Научите меня решать показательные уравнения с разными основаниями, например вот такое, я чего то туплю
Аноним 06/04/17 Чтв 23:52:54 #89 №14359 
>>14320
логарифмируешь обе части равенства
Аноним 07/04/17 Птн 00:13:45 #90 №14384 
>>14359
Тааа не палучаица
Аноним 07/04/17 Птн 05:14:21 #91 №14433 
>>14384
бля пририсуй логарифт к обеим частям и еби мне мозг.
Аноним 07/04/17 Птн 09:44:50 #92 №14446 
>>14320
Делишь обе части на 2^(5-x)
Получаешь:
1 = 4.5 * (4.5)^(5-x)
1 = 4.5^(6-x)
0 = 6 - х
х = 6
Аноним 10/04/17 Пнд 23:04:07 #93 №14612 
Мне посоветовали тут изучать Алгебру Гельфанда-Шеня. Я её скачал, начал решать задачи и у меня возникает вопрос почти до каждой. Как их решать? Авторы не дают каких-то вменяемых пояснений по своим задачам. Я худо-бедно дополз за неделю до 51 и 52 задачи, где степени. И я разрыдался, потому что нихрена там не понимаю. Как возраст Земли мне поможет узнать, сколько цифр в десятичной заиписи числа 2^20? Почему они не могут просто объяснить, какие методы мне применять при решении той или иной задачи? Я в отчаянии. Как вообще работать с этой книгой?
Аноним 11/04/17 Втр 10:46:21 #94 №14616 
.png
Какому выражению тождественно данное?
Аноним 11/04/17 Втр 12:08:07 #95 №14622 
>>14612
>Почему они не могут просто объяснить, какие методы мне применять при решении той или иной задачи?
И смысл в этом? Ты должен сам находить эти методы. Если не можешь решить, пропускай и решай другие задачи, потом возвращайся к тому что не смог решить.
Аноним 11/04/17 Втр 14:43:04 #96 №14624 
>>14229
Тащемта паркет -- охуенный метод для решения геометрических задач.
Аноним 11/04/17 Втр 22:48:03 #97 №14634 
>>14612
Ни одна из задачек Гельф-Шеня не является эвристической, есть только задачи-шутки на смекалочку.
Полностью прорешать эту брошюрку за день под силу даже пятикласснику (если не ошибаюсь, там около 400 элементарных упражнений: половина на счет, половина очевидные).
Так что, парень, если у тебя возникают такие серьезные проблемы в освоении такого простого материала, и ты еще и рыдаешь, то лучшим советом будет забить на математику вообще, это не твоё.
Ну а если сдаваться неохота, то возьми себе репетитора. С таким-то умишкой заниматься самообразованием не стоит.
Хороший педагог даже такое дно сможет вытащить.
Аноним 11/04/17 Втр 23:13:43 #98 №14635 
>>14634
Может это ты слишком умный?
Аноним 12/04/17 Срд 02:44:03 #99 №14636 
>>9338 (OP)
Всегда была интересна лестница развития математика.
В школьных программах брали всего и по чуть чуть, а как дела обстоят в специальных заведениях?
Как изучить всю математику?
Аноним 12/04/17 Срд 03:15:10 #100 №14637 
>>14636
а вот и школота подъехала
>Как изучить всю математику?
Никак. Даже если ты будешь заниматься по 2 Грота в день, то тебе все равно не хватит и всей жизни, чтобы хорошо разбираться во всех областях
>В школьных программах брали всего и по чуть-чуть
Нихуя там не брали, школьная математика - это наука древних веков, там нет ничего, что не придумали в древности, кроме ошметков матана
Аноним 12/04/17 Срд 03:56:47 #101 №14640 
>>14637
Комплексные числа в школе таки есть.
Аноним 12/04/17 Срд 03:56:55 #102 №14641 
То, что мы сегодня называем "математикой", было почти целиком выковано в XX веке
школьный предмет "Математика" Алгебра/Геометрия к нормальной математике отношения не имеет
одна только синтетическая геометрия и устный счет, от этого тянет блевать
>>14636
>а как дела обстоят в специальных заведениях
Почти также хуево. только в самых топовых школах (57, 239 и подобн) в матклассах детям дают немножко прикоснуться к некоторым совсем базовым вещам, и всё.
Кому нужно - тот ебонит сам.
Аноним 12/04/17 Срд 18:43:02 #103 №14669 
IMG0322.JPG
Посоветуйте видеозаписей лекций по теории функций комплексного переменного. Хочется поднять свой анализ с днищеуровня.
Аноним 12/04/17 Срд 20:02:44 #104 №14670 
Господа математики, есть следующая проблема оптимизации.
Система из 6 уравнений с двумя неизвестными/параметрами.
Что бы вы использовали, чтобы найти пару параметров (x,y), которые наилучшим образом с наименьшей погрешностью удовлетворяют все шесть уравнений?

Метод наименьших квадратов вручную писать это пиздец, а на работе матлаб без оптимизейшен тул бокса. С лицензией все строго.

Заранее спасибо за быстрый ответ.
Аноним 12/04/17 Срд 20:03:25 #105 №14671 
>>14670
Поправочка - уравнения линейные.
Аноним 12/04/17 Срд 21:18:49 #106 №14672 
>>14670
Возьму х точеный, выражу через у дроченый. Сам за подстановку, мать за грифики с точками пересечения
Аноним 12/04/17 Срд 21:38:36 #107 №14673 
>>14672
Ты не понял в чем суть.
Прямых будет ШЕСТЬ. И никакого пересечения у них не будет. Общего так точно. Нет решения у этой системы.
Но я уже нашел нормальный метод. Спасибо.
Аноним 13/04/17 Чтв 22:57:30 #108 №14765 
.png
КАКОГО ХУЯ 6 КОГДА 4?! БЛДЖАД МЕНЯ УЖЕ ЗАЕБАЛИ ЭТИ ДАУНСКИЕ ОШИБКИ ЭТИХ """"""""""""""""""""""""""ЭКСПЕРТОВ ЯНДЕКСА"""""""

Может у кого-то лучшая альтернатива для подготовки есть? Пиздец какой-то.
Аноним 13/04/17 Чтв 23:05:47 #109 №14767 
>>14616
Хуйня какая-то. Применяя формулу двойного угла выходит Б, но подставляя 0 и 90 градусов выходит В. Какого хуя?
Аноним 14/04/17 Птн 01:19:41 #110 №14770 
>>14765
cosa cosb - sina sinb =
(cosa cosb - sina sinb) cosb/cosb =
(cosa - sina tgb) cosb

sin a = sqrt(8/9)
cos b = -sqrt(1/5)
cosa - sina tgb =
= -1/3 - 2sqrt8 / 3
= (-1 - 2sqrt8) / 3

3sqrt(5) ((-1 - 2sqrt8) / 3) -sqrt(1/5) =
= 1 + 2sqrt8
~= 1 + 5.7
~= 6.7

Ответ: 6
Аноним 14/04/17 Птн 05:47:00 #111 №14782 
14921119111510.png
>>9338 (OP)
Перекачусь из соседнего треда, где ответа не получил.
>Аноны, поправьте меня, пожалуйста.
Чую что где-то ебанул ошибку. Может даже самую идиотскую в знаках. Но перепроверка с исследованием други способом ничего не дает(да, ответ другой, 21/10, но тут почему не сошлось?).
Где косяк?
>самофикс - у второго интеграла ограничения от 1 до 0 везде, начиная с первой строчки (а то я кое-где проебался с автозаменой)
Аноним 14/04/17 Птн 10:11:51 #112 №14787 
>>9338 (OP)
У меня очень важный вопрос.
Как переформулировать задачу, чтобы постановка вопроса была правильной?
Бросается игральная кость. Каков коэффициент корреляции между выпадением 5 и 6?
Аноним 14/04/17 Птн 10:14:39 #113 №14788 
>>14787
Бросается игральная кость. Есть ли корреляция между выпадением 5 и 6? Нет.
Аноним 14/04/17 Птн 10:58:17 #114 №14789 
>>14788
Хм, а мне сказали, что коэффициент есть и равен -1/5
Аноним 14/04/17 Птн 11:02:25 #115 №14790 
>>14770

А, понял. А как корень из двух-то посчитать, чтоб не проебаться? Не помнить же его наизусть?
Аноним 14/04/17 Птн 11:13:19 #116 №14791 
>>14790
Пусть ты извлекаешь квадратный корень из числа A. В нашем случае A=2.
1. Примерно прикидываешь, чему равен корень. В нашем случае это 1 с копейками.
2. Принимаешь x1 = 1.
3. x2 = 1/2 (x1 + A/x1) = 1/2(1+2/1) = 1.5 = 3/2
4. x3 = 1/2 (x2 + A/x2) = 1/2(3/2+4/3) = 17/12
5. x4 = 1/2 (x3 + A/x3) = 1/2(17/12+24/17) = 577/408
и т.д., пока не надоест.
Аноним 14/04/17 Птн 13:39:54 #117 №14809 
тест.PNG
>>9338 (OP)
объясните школьнику, причем здесь проценты оценок А? скуяли 23%?
Аноним 14/04/17 Птн 18:19:45 #118 №14832 
>>14791
Ну охуеть теперь. А если будет корень из трех или пяти? На каждое число помнить алгоритм что ли блядь?
Аноним 14/04/17 Птн 18:20:39 #119 №14833 
.png
.png
По какому алгоритму (НЕ ПОДСТАВЛЯЯ) можно решить левое и КАК РЕШИТЬ ПРАВОЕ БЛЯДЬ? У меня выходит график где только по две точки можно пересечь горизонтальной.
Аноним 14/04/17 Птн 23:22:17 #120 №14853 DELETED
>>14832
Ты довн? Тебе дали алгоритм для любого числа. Берешь приближение с потолка, а потом его уточняешь пока не заебет.
Аноним 14/04/17 Птн 23:24:22 #121 №14854 
>>14833
Находишь экстремумы по производной, хуле.
Аноним 15/04/17 Суб 01:00:41 #122 №14865 
>>14833
рисуй графики
Аноним 15/04/17 Суб 04:41:59 #123 №14870 
>>14832
Корень из трёх - полагаешь A=3, остальные числа не меняешь.
Корень из пяти - полагаешь A=3, x1=2, остальные числа не меняешь.

Это называется "итерационная формула Герона".
Аноним 15/04/17 Суб 22:13:25 #124 №14905 
>>14854
>>14865
Охуительная история. КОНКРЕТНЕЙ МОЖНО БЛЯДЬ?

>>14870
Что такое А?
Аноним 15/04/17 Суб 23:10:05 #125 №14907 
>>14637
>школота
Ты конечно извини, но ты немного ошибся.
Просто мне на старость лет стало интересно всё, что я проебал.

Аноним 16/04/17 Вск 03:02:06 #126 №14917 
>>14905
Целое число, из которого тебе нужно извлечь квадратный корень. Для извлечения корня ты строишь последовательность приближений по формуле xi+1 = 1/2 (xi + A/xi). Число x1 прикидываешь на основе здравого смысла.
Аноним 16/04/17 Вск 15:03:34 #127 №14932 
Какой математический аппарат надо для понимания и применения p-адических чисел?

И посоветуйте, пожалуйста, что-то по модульной арифметики (разные уровни, да?).
Аноним 16/04/17 Вск 18:31:46 #128 №14949 DELETED
КАКОГО ХУЯ ОТВЕТ В, КОГДА ПО УПРОЩЕНИЮ ЧЕРЕЗ ДВОЙНОЙ УГОЛ ВЫХОДИТ Б? ПРИ ЭТОМ ПРИ ПОДСТАВЛЕНИИ ИМЕННО ЧТО В! КАКОГО ХУЯ БЛДЖАД?!
Аноним 19/04/17 Срд 00:54:12 #129 №15435 
Здарова, прошу помочь с темой производная, так как преподша не может нормально объяснить. Я почти ничего не понимаю по этой теме, знаю только формулы, но как их правильно использовать не знаю. Надеюсь, что ответите
Аноним 19/04/17 Срд 12:45:13 #130 №15459 
fihtengoltz.jpg
>>15435
Читни Фихтенгольца.
Аноним 19/04/17 Срд 16:50:22 #131 №15476 
>>15459
Спасибо, обязательно прочитаю
Аноним 20/04/17 Чтв 12:19:18 #132 №15562 
>>15459
Открываю такой первый том "Курса..." и сразу же вижу, что в доказательстве невозможности представления sqrt2 в виде p/q пропускается уточнение, что квадрат четного числа есть число четное, и наоборот, на что обращали внимание другие авторы, приводившие данное доказательство.

Конечно, это мелочь, легко доказываемая сообразительным школьником, но год назад она бы поставила меня в тупик. Хорошо ли это или плохо, я не знаю, ведь с одной стороны, дурак вроде меня может забросить курс, а другой я потренировался бы в доказательствах простых утверждений, в случае чего перечитал бы курс элементарной алгебры или спросил бы у товарища.
Аноним 20/04/17 Чтв 15:52:40 #133 №15593 
есть множество M. из множества нужно выделить N равномощных подмножеств, |M_1| = |M_2| = ... = |M_n| и найти функцию из M в n. n - индекс подмножества, M_n.

пример:
пусть элементы M - любые натуральные числа.
N = 2 и функция f(m) : m mod(2)
f(m) = 0 если m четное, или f(m) = 1

но в M может быть больше четных элементов, чем нечетных и потому, полученные подмножества не будут равномощными, |M_1| =/= |M_0|

пусть M как в примере. с чего начать поиск такой функции?
Аноним 20/04/17 Чтв 16:15:04 #134 №15595 
>>15593
я понял, что просто с любым множеством чисел M, неполучится найти функцию.

ненужно равномощные. но в M_k не должно быть намного больше элементов чем в M_j.
элементы M, это слова из языка, Дом, Живот, ... некоторые слова, напрмер на букву А (англ.), встречаются чаще, чем другие слова.
зная про частоту слов нчинающихся на разные буквы, можно найти функцию?
sage[mailto:sage] Аноним 20/04/17 Чтв 18:33:29 #135 №15613 
>>15595
разобрался.
Аноним 20/04/17 Чтв 22:21:49 #136 №15632 DELETED
Лево: КАКОГО ХУЯ БЛДЖАД ОТВЕТ НАХУЙ 1 Б КОГДА ТАМ ДВЕ ТОЧКИ ПЕРЕСЕЧЕНИЯ СУКА БЛЯДЬ ДАУНЫ НАХУЙ ЕБАТЬ!

https://www.wolframalpha.com/input/?i=(x+-+1)%5E2+%2B+(y+-+1)%5E2+%3D+1;+y+%3D+3%5E%5Blog_3(x-1)%5D



Право: КАКОГО ХУЯ ОТВЕТ В, КОГДА ПО УПРОЩЕНИЮ ЧЕРЕЗ ДВОЙНОЙ УГОЛ ВЫХОДИТ Б? ПРИ ЭТОМ ПРИ ПОДСТАВЛЕНИИ ИМЕННО ЧТО В! КАКОГО ХУЯ БЛДЖАД?!
Аноним 21/04/17 Птн 01:19:39 #137 №15681 
>>14907
Я тут мимо, но таки скажу, что ты зря это написал. В данном случае школота - скорее уровень развития, чем возраст. Если пятиклассник задает вопрос, как выучить ВСЮ математику (физику, химию, etc) то ему это простительно, он пока не видел вообще ничего, хоть сколько-нибудь приближенного к математике. Но если человек "на старости лет" задается таким вопросом, то не очень понятно, как он сам смог компьютер включить и капчу ввести.
Аноним 21/04/17 Птн 01:40:30 #138 №15683 
.png
КАК ЭТО РЕШИТЬ БЛЯДЬ?! ДОШЕЛ ДО
> Х С ХУЕМ СВЕРХУ * 3 С КВАДРАТИЧЕСКИМ ХУЕМ СВЕРХУ = 1
ДАЛЬШЕ ЧТО БЛЯДЬ НАХУЙ?!


https://www.wolframalpha.com/input/?i=log_3(x)%5E2+%2B+(2x-5)log_3(x)+%2B+x%5E2+-5x%2B4%3D0

Вольфрам жидует на решения, только ответы выдает
Аноним 21/04/17 Птн 04:59:58 #139 №15685 
>>15683
Вероятнее всего аналитически не решается.
Аноним 21/04/17 Птн 15:39:24 #140 №15696 
>>15683
Замени логарифм на y и посчитай дискриминант получившегося квадратного уравнения. Дальше просто всё.
Аноним 21/04/17 Птн 18:47:44 #141 №15704 
>>15683
Заменяешь log_3(x) на y, получаешь такую хуйню y^2+(2x-5)y+x^2-5x+4=0 далее воспринимая заведя за скобки получишь эту хуйню (x+y-4)(x+y-1)=0 и тем самым получишь 2 уравнения которые хуй знает как решать x_1+log_3(x_1)-4=0, x_2+log_3(x_2)-1=0
Аноним 21/04/17 Птн 18:55:47 #142 №15705 
>>15683
Видел тебя на форчане.
Аноним 23/04/17 Вск 21:30:01 #143 №16077 
>>9338 (OP)
Мне еще в /sci в мат треде на просьбу дать учебник физики в котором бы шла и математика посоветовали учебник который обозвали "ПТУшным для даунов", я его давно проебал. Может подскажет кто?
Аноним 23/04/17 Вск 21:55:20 #144 №16082 
>>16077
Высшая математика для физиков и техников.
Аноним 24/04/17 Пнд 12:11:47 #145 №16114 
>>15696
Не выйдет никакого квадратного уравнения, иксы же остаются.
Аноним 24/04/17 Пнд 17:28:55 #146 №16209 
>>16077
Иродов?
Аноним 24/04/17 Пнд 18:58:01 #147 №16212 
>>16114
Относительно у квадратное.
Аноним 24/04/17 Пнд 22:00:26 #148 №16232 
>>16077
Савельев?
Аноним 24/04/17 Пнд 23:01:54 #149 №16243 
>>16212
И дальше что? С иксами что делать?
Аноним 25/04/17 Втр 02:04:11 #150 №16255 
>>16243
Сумма квадратов же.
(log3(x) + x - 2.5)^2 = 2.25
Получаешь два уравнения:
log3(x) = 4 - x
log3(x) = 1 - x
Слева возрастающая функция, справа убывающая, если подберешь решение, то оно будет единственным. 3 и 1
Аноним 25/04/17 Втр 18:39:04 #151 №16304 
>>16255
> (log3(x) + x - 2.5)^2 = 2.25
Где ты это взял?

Я понимаю еще log3(x)(log3(x) + 2x - 5) = -(x-1)(x-4)

И так можно подбором найти 1 и 3, но во-первых это слабо связано с твоей ерундой и во-вторых все еще неизвестно есть ли другие корни.
Аноним 25/04/17 Втр 20:22:43 #152 №16306 
>>16304
Ты не можешь элементарные преобразования провести?
log3(x)^2 + (2x-5)log3(x) + x^2-5x+4 = 0
log3(x)^2 + 2(x - 2.5)log3(x) + (x - 2.5)^2 - 2.25 = 0
Квадрат суммы.
(log3(x) + (x - 2.5))^2 - 2.25 = 0
Аноним 25/04/17 Втр 20:35:45 #153 №16307 
>>16306
Это КВАДРАТ СУММЫ а не наоборот.

Ну и дальше что? Как ты отсюда берешь

> log3(x) = 4 - x
> log3(x) = 1 - x
Аноним 25/04/17 Втр 21:25:49 #154 №16309 
> При каком наименьшем значении параметра a неравенство ax^2-2x + a ≥ 0 справедливo для всех x∈R?

Я просто посчитал а при нулевом дискриминанте и вышел правильный ответ, но ПОЧЕМУ ЭТО РАБОТАЕТ?
Аноним 26/04/17 Срд 02:15:26 #155 №16321 
>>16307
sqrt(2.25)=1.5
Аноним 26/04/17 Срд 02:16:24 #156 №16322 
>>16309
Если D<=0 то полином >= 0
Аноним 26/04/17 Срд 16:20:35 #157 №16357 

Аноны, очень хелп плез.

Как вы знаете есть задачки коммивояжёра:
Имеем 20 точек и пути между ними. четное кол-во путей из каждой вершины.
Начав из точки А в нее же и вернемся, пройдя все 19 чекпойнтов самым выгодным маршрутом.

Моя задачка: есть 40 точек и пути между ними.
Начинаем в А, требуется пройти любых 29 чекпойнтов самым выгодным маршрутом. 10 вершин должно остаться без внимания.

Куда читать? Где могут быть такие примеры?
диплом строится на этом, если есть фланеры-энтузиасты могу расписать подробнее, но вряд ли оно надо
Аноним 27/04/17 Чтв 11:46:42 #158 №16421 
>>16357
метод ветвей и границ гугли
Аноним 27/04/17 Чтв 16:45:15 #159 №16436 
>>16421
тогда путь будет состоять из всех вершин т.е. из 40
Аноним 28/04/17 Птн 01:24:30 #160 №16486 DELETED
>>16322
Но тогда наименьшего значения не существует блядь нахуй.
Аноним 28/04/17 Птн 02:42:48 #161 №16490 DELETED
Материальная точка движется вдоль оси абсцисс по закону x (t) изменения координаты x от времени t. Для которого из случаев скорость материальной точки возрастает со временем?

КАКОГО ХУЯ ПЕНИСА БЛЯДЬ ШЛЮХА И Г, И Д ПОДХОДИТ СУКА САМКА СОБАКИ!
Аноним 28/04/17 Птн 18:48:45 #162 №16525 DELETED
>>16490
В Г сначала убывает, а потом только возрастает.
Аноним 28/04/17 Птн 18:54:55 #163 №16526 
>>16321
Это дает
> log3(x) = 4 - x

А это где ты взял блджад?
> log3(x) = 1 - x

И где там возрастающая, а где убывающая функции тоже ничего не ясно.
Аноним 28/04/17 Птн 19:37:07 #164 №16528 
.dib
НАЙТИ КОЛИЧЕСТВО ЭКСТРЕМУМОВ.

НУ ПРОИЗВОДНАЯ ВЫДАЕТ ДЖВА НУЛЯ! А ОТВЕТ 5. КАКОГО ?!
Аноним 28/04/17 Птн 20:04:25 #165 №16531 DELETED
> НАЙТИ НАИБОЛЬШИЙ ОТРИЦАТЕЛЬНЫЙ КОРЕНЬ.
> @
> В ОТВЕТЕ МЕНЬШИЙ ИЗ ДВУХ
Аноним 28/04/17 Птн 20:09:06 #166 №16532 DELETED
> А Случайное событие: в результате деления двух натуральных чисел получили ноль.
> Б Случайное событие: в результате деления двух натуральных чисел получили рациональное число.
> В Случайное событие: в результате деления двух натуральных чисел получили иррациональное число.
> Г Невозможна событие: в результате деления двух натуральных чисел получили отрицательное число.
> Д Вероятная событие: в результате деления двух натуральных чисел получили правильную дробь.

Б И Д ПОДХОДЯТ, В ЧЕМ ПРОБЛЕМА ДАУНОВ?
Аноним 28/04/17 Птн 20:23:01 #167 №16533 
>>16526
sqrt(2.25)=+-1.5
Аноним 28/04/17 Птн 20:40:40 #168 №16535 
>>16528
ну у производной пять нулей, всё в порядке
Аноним 28/04/17 Птн 21:05:58 #169 №16541 DELETED
>>16532
Зарепортил все твои посты. Добротред не для этого создавался. Ты злой!
Horen !!htiXWTUYyY 28/04/17 Птн 21:19:16 #170 №16545 DELETED
>>16541
А я злой?
Аноним 28/04/17 Птн 21:59:16 #171 №16553 DELETED
>>16545
У тебя мозг разрушен. Ты не специально.
Horen !!htiXWTUYyY 28/04/17 Птн 22:05:33 #172 №16554 DELETED
>мозг разрушен
Докажи.
Аноним 29/04/17 Суб 03:06:10 #173 №16567 
Сап, пацаны. Короче я задумал замутить генератор криптоалгоритмов. Все бы ничего, задача довольно простая (по крайней мере в случае с симметричными алгоритмами), но вот с алгоритмами с открытым ключем - я не понимаю как все происходит. В каком направлении надо курить, чтобы это сделать? Суть такова, я хочу сделать генератор, который генерирует случайный криптоалгоритм, у которого на шифрование/расшифровку одного килобайта уходит не менее секунды при нагрузке 1 ядра моего проца (phenome II x6 1095) на 100%. Что учить надо для этого? Просто все современные книги по криптографии - полная хуйня. Они описывают лишь уже существующие криптоалгоритмы и расписывают лишь то, что вот это системы с закрытым ключем, это с открытым и все.
Аноним 03/05/17 Срд 14:48:37 #174 №16988 
tumblrng727kaDGd1s4fz4bo1500.gif
Здравствуйте! Помогите, пожалуйста, с заданием.

При регистрации в компьютерной системе каждому пользователю выдаётся пароль, состоящий из 9 символов. Из соображений информационной безопасности каждый пароль должен содержать хотя бы 1 десятичную цифру, как прописные, так и строчные латинские буквы, а также не менее 1 символа из 6-символьного набора: «&», «#», «$», «», «!», «@». В базе данных для хранения сведений о каждом пользователе отведено одинаковое и минимально возможное целое число байт. При этом используют посимвольное кодирование паролей, все символы кодируют одинаковым и минимально возможным количеством бит. Кроме собственно пароля, для каждого пользователя в системе хранятся дополнительные сведения, для чего выделено целое число байт; это число одно и то же для всех пользователей.
Для хранения сведений о 20 пользователях потребовалось 500 байт. Сколько байт выделено для хранения дополнительных сведений об одном пользователе? В ответе запишите только целое число – количество байт.
Примечание. В латинском алфавите 26 букв.
Задание 13 № 11349
Пояснение.
Согласно условию, в пароле могут быть использованы 10 цифр (0..9), 52 буквы (строчные и прописные) и ещё 6 символов из спецнабора, всего 10 + 52 + 6 = 68 символов. Известно, что с помощью N бит можно закодировать 2N различных вариантов. Поскольку 26 < 68 < 27, то для записи каждого из 68 символов необходимо 7 бит.
Для хранения всех 9 символов номера нужно 9
7 = 63 бит, а т. к. для записи используется целое число байт, то берём ближайшее не меньшее значение, кратное восьми, это число 64 = 8 * 8 бит (8 байт).
Для хранения всех сведений об одном пользователе используется 500/20 = 25 байт, следовательно, для хранения дополнительных сведений выделено 25 − 8 = 17 байт.

Ответ: 17.


Я считаю, что в решении напрочь забыли о таких вещах как "не менее 1 цифры, одного из 6 символов" в пароле. Т.е если у нас на каждым из девяти символов может быть любое из 68 чисел, то есть хотя бы один вариант, противоречащий условию - когда все девять символов являются любыми из 52 букв, без цифр и тех 6 символов. И эти варианты в решении бодро причислили к верным( а ведь нам нужен наименьший объём). Я никак не могу понять как считать количество информации в таких случаях, выходит вообще решение олимпиадного уровня.

Заранее спасибо.
Аноним 03/05/17 Срд 22:30:42 #175 №17030 
> Найдите медиану упорядоченной выборки первых девяти целых чисел из области значений функции
Что это значит?
Аноним 04/05/17 Чтв 12:24:30 #176 №17129 
>>17030
Берёшь мн-во значений функции, берёшь первые 9 целых чисел оттуда, находишь их среднее арифметическое.
Аноним 04/05/17 Чтв 16:40:52 #177 №17181 
.png
>>17129
Что такое "первые целые числа" и каким образом выходит 0?
Аноним 04/05/17 Чтв 21:09:27 #178 №17229 
>>16988
если я правильно понял условие, то ты прав и решение неправильное. Я решал бы так
Начал бы сначала с спец.символов -> С69 = 9!/((9-6)!*6!)
Вот
а потом уже все остальные перечисляешь( ну, тупо умножаешь)
решение не сильно изменится, но ответ будет меньше(каюсь, я не считал циферки)
Аноним 04/05/17 Чтв 21:12:11 #179 №17230 DELETED
>>16988
Бл, тебе же рассказали решение в треде ЕГЭ по инфе О_О
Аноним 07/05/17 Вск 19:37:45 #180 №17447 
.png
ПОМОЩЬ
Аноним 07/05/17 Вск 21:05:38 #181 №17448 
>>17447
Найди производную, приравняй к нулю ... профит
Аноним 07/05/17 Вск 21:06:36 #182 №17449 
>>17448
Понял уже. Долго вы тут.
Аноним 08/05/17 Пнд 18:18:25 #183 №17480 
Как на английском называется "финально ограниченная функция" (у зорича так)?
finallly bounded не катитj
Аноним 08/05/17 Пнд 18:39:33 #184 №17483 
>>17480
А тебе зачем, если не секрет?
Обычно интересуются обратным переводом, с англ. на русский.
Аноним 08/05/17 Пнд 19:03:01 #185 №17484 
>>17483
Я просто не понимаю, что он имеет в виду, это его личный термин. Значит есть ограниченная функция, а есть финально ограниченная...
Аноним 08/05/17 Пнд 19:22:31 #186 №17485 
>>17480
Никак не называется, это его оригинальный термин.

>>17484
Пусть f определена на X и принимает значения в Y, M - непустое подмножество X. Символом f(M), как обычно, обозначается множество всех таких y из Y, что существует m из M такое, что f(m)=y.

Пусть Y - метрическое пространство.
Функция f называется ограниченной на множестве M, если f(M) целиком содержится в каком-нибудь шаре в Y.

Пусть B - база множеств в X.
Функция f называется финально ограниченной по базе B, если существует множество M из базы B такое, что f ограничена на M.
Аноним 08/05/17 Пнд 19:47:53 #187 №17488 
>>17485
>Символом f(M), как обычно, обозначается множество всех таких y из Y, что существует m из M такое, что f(m)=y.
Юмор какой, ты еще знакосочетания определи.
Аноним 08/05/17 Пнд 20:08:30 #188 №17490 
14754681908970.jpg
>>17488
Аноним 15/05/17 Пнд 18:26:16 #189 №18069 
>>13453
AO же, это восьмой класс или девятый?
Аноним 17/05/17 Срд 13:39:32 #190 №18422 
Телочка.jpg
>когда открыл для себя введение к книжке и там сказано, что "средний студент прочитает её за 2 дня", а ты читаешь уже месяц.
Аноним 18/05/17 Чтв 16:03:55 #191 №18537 
>>18422
Вавилов?
Аноним 18/05/17 Чтв 16:46:58 #192 №18544 
Screenshot2017-05-1819-42-37.png
>>18537
Нет.
Аноним 19/05/17 Птн 07:31:07 #193 №18601 
>>17181

ПОМОГИТЕ
Аноним 19/05/17 Птн 07:41:04 #194 №18603 
.png
.png
КАК
Аноним 20/05/17 Суб 20:22:44 #195 №18727 
>>18603
ПИФАГОР
Аноним 21/05/17 Вск 04:41:56 #196 №18757 
>>18601
Тебя наебали. Медиана это не среднее арифметическое.медиана — это такое число выборки, что ровно половина из элементов выборки больше него, а другая половина меньше него. Нужно упорядочить выборку, затем взять средний элемент.
Аноним 24/05/17 Срд 21:03:40 #197 №19123 
14956356128510.png
jackie-chan-meme-why-620x4961.jpg
Как? Предел этого логарифма, в отрыве от x2 при x → ∞ равен 0 же? Тогда по какой логике производятся дальнейшие действия?
Сколько ни смотрел свойства бесконечно малых/больших, но в итоге ничему не научился оттуда. В каком порядке вычислять? Где применять те или иные свойства? Дурдом.
Аноним 24/05/17 Срд 21:15:23 #198 №19126 
>>19123
Своди к первому замечательному пределу.
Аноним 25/05/17 Чтв 01:02:31 #199 №19134 
>>19123
x^2 log (1 - 3/(x^2 + 4)) ~ -3x^2/(x^2+4)
Аноним 25/05/17 Чтв 17:43:23 #200 №19158 
>>19134
ты бы ещё написал -3/1
Аноним 26/05/17 Птн 19:12:45 #201 №19181 
>>18757
Это я знаю, но что такое ПЕРВЫЕ 9 ЦЕЛЫХ ЧИСЕЛ? Что это значит блядь? Ведь я не знаю какие первые числа области значений сами по себе, потому что там икс, соответственно сначала надо подставить под него что-то. Но что? Ну вот я пробовал -4 по 4, 0 по 8, все, что только можно придумать - нихуя не 0, а это ответ.
Аноним 27/05/17 Суб 09:34:32 #202 №19200 
P70527-092846(1).jpg
Помогите, пожалуйста, 2 день не могу решить
Аноним 27/05/17 Суб 10:03:49 #203 №19202 
>>19200
я решил
Аноним 27/05/17 Суб 19:43:03 #204 №19214 
Надо привести квадратичную форму к каноническому виду методом собственных векторов (сначала найти собственные вектора матрицы квадратичной формы). Но характеристический многочлен матрицы имеет корни, в формуле которых есть корень третьей степени (не упрощается). ЧЯДНТ?
Аноним 27/05/17 Суб 20:01:53 #205 №19216 
>>19200
умножь нижнее уравнение на 3, дальше все легко
Аноним 27/05/17 Суб 21:01:17 #206 №19229 
.png
КАК?! Скаляры АБ - 10, АД - 24. Так какого тут выходит что их квадраты не 100 и 24^2? Как вообще тут можно квадрат 24 найти? Это столбом что ли унижаться?
Аноним 30/05/17 Втр 11:25:23 #207 №19379 
Анончики, вопрос такой: допустим, дана сфера, на ее поверхности находится некоторое количество точек, каждая точка имеет координаты(x,y,z) и массу. я отмечаю на сфере область, в которой находится сколько-то точек. как мне найти центр масс для этой области?
п.с. найденный центр масс должен находиться на поверхности сферы, не внутри ее
Аноним 30/05/17 Втр 14:09:43 #208 №19384 
w4ruDD9IXOs.jpg
помогите с матлогикой хоть какими-то задумками пожалуйста. Интересуют два последних номера
Аноним 30/05/17 Втр 14:49:22 #209 №19386 
>>19384
Я сам не шарю, но в предпоследней же просто надо построить 2, 1/2 через данные фигни? игра-конструктор получается.
тебе получается дано отношение меньше, которое в себе содержит равно, так тогда определи через них сложение или там функцию следования S(a)=a+1(наоборот из формулы a>b <=> существует c(a=b+c) и a=/=b), потом определи двойку(например, как единственный элемент R, такой, что S(1)=2, потом каким-то еще трюком введи 1/2, а там уже какую хочешь формулу составляй.
Аноним 30/05/17 Втр 15:12:26 #210 №19388 
>>19379
>п.с. найденный центр масс должен находиться на поверхности сферы, не внутри ее
Если бы так было, можно было бы провести через центр масс касательную к сфере плоскость: все материальные точки а твои точки подходят под это определение окажутся с одной стороны от плоскости, в то время как центр масс - в плоскости. Такое невозможно достаточно ввести нормальную к плоскости ось X с нулем в плоскости и посчитать Х-координату центра масс: она получится положительной.
Аноним 07/06/17 Срд 14:52:20 #211 №19869 
>>19388
Наверное, он имеет в виду, что ему надо найти проекцию центра масс на поверхность сферы.
Аноним 10/06/17 Суб 17:02:49 #212 №20026 
Пацаны, по какому учебнику можно угареть по теории множеств?
Аноним 15/06/17 Чтв 08:26:33 #213 №20213 
>>19384

Где ты учишься?

x \in {1/2,2} <=>
not((1+1)x<1 or 1<(1+1)x) or not(x<1+1 or 1+1<x)

-это номер 2.
Что подразумевается под понятием "система"? Просто класс множеств?
Аноним 15/06/17 Чтв 08:38:42 #214 №20214 
ну или

x \in {1/2,2} <=>
not(x+x<1 or 1<x+x) or not(x<1+1 or 1+1<x)
Аноним 15/06/17 Чтв 08:43:46 #215 №20215 
Первая сверху - нет, не является тождественно истинной, так как контрпример - два натуральных ряда с общим началом.
Аноним 15/06/17 Чтв 08:48:47 #216 №20216 
хоть анону (>>19384 ) уже, вероятно, не нужен ответ - но мне интересна задача №3.
Аноним 16/06/17 Птн 21:57:00 #217 №20335 
>>20026
Не нашёл ничего понятнее(для меня, офк) чем metamath-овскую брошюрку.
Есть разные очень солидные тома, но они не давали нужного для моего понимания строгости выкладок.

Расширяя дискуссию: у меня такое чувство, что пруфассистанты особенно нужны тем, кто плохо понимает человеческие объяснения.
Аноним 17/06/17 Суб 04:23:40 #218 №20355 
>>20026
Очевидный Мендельсон.
Аноним 27/06/17 Втр 19:33:04 #219 №21071 
matrix.png
Допусти, дана матрица с данными на картинке, как её привести к каноническому виду?
Аноним 27/06/17 Втр 23:47:33 #220 №21084 
>>21071
для начала найти собственные числа и собственные вектора.
Аноним 28/06/17 Срд 21:41:15 #221 №21119 
>>9338 (OP)
анон нужна помощь, подскажи какие скачать учебники по математике начиная с 5го класса, в школе страдал хуйней в 8 классе было вроде не плохо , дальше скатился в сраное говно ибо ленился и списывал, в универе только на первом курсе была вышка , кое-как сдал, сейчас хочу восполнить знания , задачи научится решать просто для себя , по геометрии вообще пиздец был можно сказать что двойка. Пока писал от стыда аж вспотел. Аноны выручайте с чего начать, какие книги можно скачать на планшет что бы теорию учить и тут же задачки и примеры решать. Думаю что нужно класса с 5го начинать и дальше до 11. Прошу помощи
Аноним 28/06/17 Срд 22:07:00 #222 №21120 
>>21119
ну дык качай школьные учебники, читай теорию, решай задачи, если что не понятно - иди на ютаб, там полно курсов, если что здесь спрашивай. Я вот тоже начал вышку учить, потихоньку продвигаюсь, вроде пока получается.
Аноним 28/06/17 Срд 22:18:08 #223 №21126 
>>21120
да их там много всяких, можешь подскажешь какие получше
Аноним 28/06/17 Срд 22:43:40 #224 №21135 
>>21126
ну у нас такие были, задачник зеленый, а учебник красный.
https://slovo.ws/urok/algebra/08/007/cover_big.jpg
Аноним 28/06/17 Срд 22:46:49 #225 №21136 
>>21126
а вообще есть канал на ютабе MathTutor, там выбирай нужный плейлист и смотри.
Аноним 28/06/17 Срд 22:52:12 #226 №21138 
>>21126
Можешь взять книги Киселева А.П.
Аноним 29/06/17 Чтв 07:46:37 #227 №21149 
>>21136
> MathTutor
добра тебе анон
Аноним 29/06/17 Чтв 07:47:22 #228 №21150 
>>21138
на работу приду поищу
Аноним 04/07/17 Втр 13:21:49 #229 №21277 
Проверьте, пожалуйста, мои суждения по следующим задачам.
1) "Построить ограниченное множество вещественных чисел, имеющее ровно три предельные точки. " Я думаю, что треугольник.
2) "Построить компактное множество вещественных чисел, множество предельных точек которого счетно." Думаю, что интервал .
3) "Пусть E''= Е∪Е', где Е' множество предельных точек E. Доказать, что Е'' всегда замкнуто и что E''⊂F, если Е⊂F и F замкнуто." Почему F должно быть замкнутым?
Аноним 04/07/17 Втр 13:55:41 #230 №21278 
>>21277
Треугольник - это подмножество R^2. А тебя просят указать подмножество R.
Аноним 04/07/17 Втр 14:37:02 #231 №21279 
>>21277
>Думаю, что интервал
Интервал не является компактным множеством.
Аноним 04/07/17 Втр 20:52:59 #232 №21291 
>>21279
Но он же открытый.
Аноним 04/07/17 Втр 20:53:26 #233 №21292 
>>21278
Извини, немного не понял, почему подмножество R?
Аноним 04/07/17 Втр 21:06:41 #234 №21293 
>>21292
>ограниченное множество вещественных чисел
>множество
>вещественных чисел
Аноним 04/07/17 Втр 22:21:59 #235 №21311 
>>21293
Теперь понял. Ладно. Какой будет ответ?
Подойдёт множество из трёх точек?
Аноним 05/07/17 Срд 11:24:02 #236 №21338 
>>21291
(a,b) - не является компактом в R
[a,b] - компакт в R
впрочем тут надобно спросить что ты понимаешь под компактностью.
Аноним 05/07/17 Срд 13:43:00 #237 №21358 
>>21338
Не, это я затупил. Перепутал сегмент с интервалом.
Аноним 05/07/17 Срд 13:54:10 #238 №21359 
>>21311
Не подойдёт, потому что у него вообще нет предельных точек.
Аноним 05/07/17 Срд 13:58:29 #239 №21360 
>>21311
а ответ - ну возьми три какие-нибудь последовательности точек отрезка [0;2], которые сходятся соответственно к нулю, к 1 и к 2, и объедини их.
Аноним 05/07/17 Срд 18:29:06 #240 №21374 
>>21360
О, спасибо, кажется начал понимать. Кст, это правда, что вариантов выборки в твоём ответе 2?
Аноним 05/07/17 Срд 19:42:26 #241 №21375 
>>21374
Что?
Аноним 06/07/17 Чтв 06:36:51 #242 №21392 
>>21375
2 варинта ответа: объединение последовательности, сходящихся к 0, 2 и к 1 слева и справо.
Аноним 06/07/17 Чтв 09:36:42 #243 №21397 
>>21392
Нет. Последовательностей, сходящихся к 0, бесконечно много. Последовательностей, сходящихся к 1 и 2, тоже бесконечно много. Разных вариантов ответа - бесконечно много.
Немного переделанная задача коммивояжёра аноним 07/07/17 Птн 18:36:02 #244 №21452 
Здравствуйте
Суть проблемы: мне надо решить задачу коммивояжёра но в отличии от обычного варианта этой задачи в моём случае я могу возвращаться в уже пройденные вершины( гамильтонов цикл не обязателен но если он является самым оптимальным то почему бы и нет)
В конечном варианте я хочу сделать программу которая будет находить оптимальный маршрут для работы курьера ( курьер будет выезжать из пункта выдачи проходить по всем местам и возвращаться обратно)
самое сложное как по мне это то что я не знаю как это правильно загуглить ибо если бы я решал обычную задачу коммивояжёра то у меня уже есть 100500 готовых вариантов которые я тупо могу скопировать и вставить но в моей вариации так не выходит
Задача на теорию множеств Аноним 19/07/17 Срд 11:31:18 #245 №22132 
Пусть X - пространство всех рациональныъ чисел d(p,q)=|p-q| и E - множество всех рациональных p, таких, что 2<p^2<3. Показать, что E замкнуто и ограничено, но не компактно.
Допустимо ли такое решение: Во-первых, множество E - ограничено, так как если квадрат каждого элемента ограничен, то и сам элемент ограничен. Во-вторых, оно замкнуто, потому что X-пространство, а в нём можно выбрать замкнутый шар в силу d(p,q)=|p-q|.Тип показал, что E замкнуто и ограничено. Теперь перехожу к до-ву, что не компактно. Выберем множество: {1/n+2^(1/4),sqrt(3)|n $ N}. Из него нельзя выбрать конечного подпространства. Следовательно, E - некомпактное множество.
Аноним 20/07/17 Чтв 11:43:11 #246 №22184 
Привет, народ.
С чего бы начать изучать математику,человеку который вообще хуй соброжает в ней.
Спасибо
Аноним 20/07/17 Чтв 11:50:44 #247 №22185 
>>22184
С теории множеств.
Аноним 20/07/17 Чтв 14:37:38 #248 №22221 
>>22184
Оснований Математики, особенно конструктивизма.
Аноним 20/07/17 Чтв 14:41:02 #249 №22228 
>>22184
Здесь надо поставить вопрос, а чего ты хочешь.
В любом случае лучше всего начинать с теории множеств.
Аноним 20/07/17 Чтв 18:46:49 #250 №22249 
>>22221
Поподробней?
Другой, нетралль
Аноним 20/07/17 Чтв 21:57:08 #251 №22288 
>>21452
Используй динамику по подмножествам. Если ты хочешь расширить мн-во А вершиной b, тебе придется сначала обойти некоторое подмн-во А, пройти через новую вершину b, и наконец допройти остаток мн-ва А. При всех хождениях придётся использовать кратчайшие пути между вершинами, которые ты предварительно найдешь Флойдом. Работать будет долго, но зато точно.

Писал на коленке, скорее всего обосрался
Аноним 21/07/17 Птн 23:55:18 #252 №22319 
>>22184
>>22185
>>22221
>>22228
Это такой самотроллинг? Почти одновременные посты
Аноним 22/07/17 Суб 02:40:20 #253 №22322 
Здравствуйте, я тут на днях решил повторить ангем, который ещё на первом курсе был. Почитал учебник Александрова, Геометрия. Так вот, там есть задача.
докажите, если на поверхности с двумя семействами прямолинейных образующих есть прямая, им не принадлежащая, то поверхность - плоскость.
Я как-то засторопился на ней. Пробовал доказать аналитически, но там какая-то куча коэффициентов вылазит. Или же частные случаи получаются. А какое-то ясное полное доказательство не приходит в голову.
Хотел попросить помощи.
Аноним 29/07/17 Суб 23:25:07 #254 №22574 
>>22132
Уже 10 дней прошло.
Аноним 30/07/17 Вск 16:54:08 #255 №22589 
Дяденьки, научите меня евклидовой геометрии.
Школьный проект 11 класс Аноним 31/07/17 Пнд 00:20:49 #256 №22613 
pQ6AauJtqGw.jpg
Сап, тред, я хочу сделать классный исследовательский проект по математике и/или физике, чтобы он меня чему-то научил и было не стыдно его защищать, сам я ничего придумать не смог (абсолютно не имею идей. то, что обычно предлагается в школе очень ущербно, а то, что я сам придумываю скорее похоже на повторение теории, хоть и не входящей в школьный курс) и прошу помощи и советов у местных мыслителей. С меня как всегда - почёт и уважение.
Я физмат 11 класс, планирую поступать на астронома в УрФУж;имею некоторое представление об мат.анализе и некоторых других сферах математики, но в них моё представление менее серьёзно; плохо дружу с геометрией вообще не помню, когда серьёзно ею занимался, но когда занимался она давалась мне достаточно легко, возможно сейчас всё иначе; больше люблю теоретическую часть деятельности (посчитать, подумать, объяснить почему именно так и тд), но не, когда нужно что-то паять, экспериментировать и тд, хотя это не слишком страшно; с самого детства хотел стать астрономом для меня это задача №1, как стать Хокаге. Больше не могу придумать уточняющих слов, если всё-таки нужно ещё - спрашивайте. Хотел начать проект летом, но там уж как получится.
Заранее благодарю за помощь и всё такое
Аноним 31/07/17 Пнд 00:40:49 #257 №22614 
>>22613
математические методы в астрономии, астрофизика, геометрия ночного неба
https://www.youtube.com/watch?v=yxtV2HBnEbo
Интегралы и дифференциалы Аноним 09/08/17 Срд 15:04:06 #258 №22949 
trwhwrth.png
Во общем я сейчас изучаю анализ Фурье, и у меня есть несколько вопросов на тему "Интегрирование". Сразу предупреждаю, что бы не бомбили, я закончил только 9-ый класс и я все эти темы изучал самостоятельно, поэтому я могу не знать какие-то нюансы которые объясняют в шкалке. Так вот вопросы:
1) Если идёт сложение разных интегралов, то можно не писать dx по сто раз, а вынести его за скобку? Если нельзя, то почему?
2) Я знаю как интегрировать по частям и как преобразовывать дифференциалы, можете посоветовать ещё способов для интегрирования?
3) Ну ещё можете покидать ссылки на полезные ресурсы на эту тему, я просто ещё не до конца по настоящему понимаю значение дифференциала dx.

Заранее спасибо.
Аноним 11/08/17 Птн 13:24:27 #259 №23002 
>>22949
dx показывает, по какой переменной ты интегрируешь. Например, Int xy dx = x^2 y / 2, а Int xy dy = x y^2 / 2. Улавливаешь? И да, выносить за скобки нельзя, это составная часть интеграла.
Аноним 11/08/17 Птн 16:42:36 #260 №23007 
>>23002
Помню в школе сказал, что dx это просто показатель по какой переменной происходит интегрирование. Надо мной весь класс смеялся, включая учителя, обидно было
Аноним 11/08/17 Птн 20:38:57 #261 №23017 
>>23007
Но ведь ты был прав.
Аноним 12/08/17 Суб 16:58:35 #262 №23043 
>>22949
Дело в том, что интегрируются на самом деле не функции, а дифференциалы. Дифференциал - это некий объект, который можно записать в виде f(x)dx - произведения функции и dx. Тут я должен сказать, что такое dx, т.е. ответить на вопрос 3), но я не буду. Это уже наука, а не просто "правила как интегрировать"

Тем не менее, смотреть на dx просто как на указание о том, по какой переменной интегрируешь, мне кажется, для души вредно.
Аноним 12/08/17 Суб 17:02:31 #263 №23044 
>>23043
чтобы не очень наводить тумана, я всё-таки выражусь точнее. Правильно надо было сказать

Дело в том, что интегрируются на самом деле не функции, а дифференциальные формы
Аноним 13/08/17 Вск 12:21:14 #264 №23060 
>>22949
>3) Ну ещё можете покидать ссылки на полезные ресурсы на эту тему, я просто ещё не до конца по настоящему понимаю значение дифференциала dx.
d это корень из 0.
Аноним 13/08/17 Вск 17:29:07 #265 №23065 
Аноны хелп! Какая есть теория по прямой, проходящей через центр вписанной в треугольник окружности и параллельной его стороне(треугольника)?
Аноним 13/08/17 Вск 21:00:49 #266 №23077 
>>23060
Да я слышал про это, что "d это корень из 0, но d неравно 0". Это кажется Ферма придумал.
Я имел ввиду вдруг кто-то знает отличный учебник на эту тему, без воды.
Аноним 14/08/17 Пнд 18:05:39 #267 №23216 
Посоветуйте книгу по математике для первого студака первого курса с задачами офк. Закончил высшую шарагу как инженер, а по матеше знаний хуй да нихуя, хотя в начальной-средней школе по матеше чето дрочил.
Аноним 14/08/17 Пнд 18:08:13 #268 №23217 
>>23216
С. И. Шварцбурд, О. С. Ивашев-Мусатов. Алгебра и начала
анализа. Учебное пособие для ПТУ.
Аноним 14/08/17 Пнд 18:16:32 #269 №23218 
>>23217
Спс.
Аноним 15/08/17 Втр 10:03:33 #270 №23231 
Не могу разобраться с тензорами. Тут можно поспрашивать или мне в отдельный тред?
Аноним 15/08/17 Втр 12:36:32 #271 №23236 
>>23231
Спрашивать можно хотя я всё равно не отвечу, но сначала предупреждение: ни в коем случае не пытайся там ничего осознать через координаты ("компоненты").
Аноним 15/08/17 Втр 15:47:53 #272 №23244 
>>23236
Да, я понял, что целью было развить такую запись, при которой можно было бы не привязываться к какой-то определенной координатной системе, потому что вроде как разрабатывалось это для работы с криволинейными пространствами, в которых происходит какой-то леденящий душу пиздец. Но как я понял, все равно вот этот набор верхних и нижних индексов (контр- и ковариантных) описывает преобразования между системами.

Читал вот эту штуку http://www.gptelecom.ru/Articles/tensor.pdf дочитал с трудом страницы до 10ой, а потом сразу же обосрался. Начинал так же читать брошюру от NASA https://ntrs.nasa.gov/archive/nasa/casi.ntrs.nasa.gov/20020083040.pdf примерно с тем же результатом.

Короче даже не знаю, хочется какого-то более "очеловеченного" объяснения что это и как с этим работать. Читал статьи более общего характера, но в них опускаются все эти преобразования, свертка и пр.

Я вот все как-то раньше думал, что если проявить достаточно упорности и потратить время - все станет ясно. Но хер там был. Векторную алгебру хорошо знаю, с матрицами умею работать (хотя не могу сказать, что везде понимаю саму природу операций), а вот с тензорами вообще жопа.
Аноним 15/08/17 Втр 18:22:23 #273 №23249 
>>23244
Не очень понятно, что ты хочешь от этих тензоров? Если у тебя "криволинейные координаты", т.е. тебе нужна дифференциальная геометрия, то хорошая книга J. Lee - Introduction to Smooth Manifolds. Ещё, помнится, у Стернберга в первой главе Лекций по дифф. геом. было хорошо написано. В обоих книгах, помнится, есть и координаты и универсальные свойства.

Если тебе нужно тензорное произведение как алгебраическая операция, то есть множество источников, и всё гуглится. В том числе листочки из вышки, нму.

Может быть, тебе рано ещё тензоры изучать, а надо как раз гладкие многообразие, касательные расслоения, вот это всё
Аноним 01/09/17 Птн 04:17:37 #274 №24116 
312314342345345.jpg
>поэтому 3 х Г должно оканчиваться на 5.

Не пойму логики, почему вдруг "ПОЭТОМУ" оно должно оканчиваться на 5?
Аноним 01/09/17 Птн 12:25:40 #275 №24122 
>>24116
Потому что сумма разряда единиц 3xГ и перенесённой двойки равна 7.
Аноним 03/09/17 Вск 20:44:05 #276 №24263 
>>23244
Не еби мозг тензорами. Относись к ним пока как к матрицам: вот есть тензор как таблица чисел, можо умножать только ко- на контр-. Забей. Когда будешь теорию Групп изучать - поймешь что такое тензор.
Maxima Аноним 03/09/17 Вск 21:18:18 #277 №24264 
2017-09-03 211653.png
Юзаю wxMaxima, точнее, не юзаю, ибо она на любой мой запрос просто перепечатывает мой ввод.
Как заставить её реально дать ответ, а не просто красиво вывести?
Аноним 04/09/17 Пнд 20:07:23 #278 №24308 
15044671763390.png
Добрый вечер, Господа!

Есть Один Предел: lim(n->oo)(((n!)^(1/n))/n)
Интернет говорит, что ответ здесь 1/е, но решения не даёт. Сам решить не смог.

Желательно не использовать правило Лопиталя, и вообще решить как можно проще. Но рад буду любому решению, и даже совету, кроме совета идти в школу.
Аноним 05/09/17 Втр 03:03:22 #279 №24328 
>>24308
Формулу стирлинга ебани, братка
Аноним 05/09/17 Втр 11:30:57 #280 №24331 
>>24308
потенцируешь по e, затем применяешь совет >>24328
Аноним 06/09/17 Срд 01:54:05 #281 №24351 
>>24308
решил, нет?
Аноним 08/09/17 Птн 03:53:00 #282 №24438 
>>9338 (OP)
Кто нить объясните плзи, почему в совместных вероятностях, когда 2 события формула вычитает их одновременное появление:
А+В=Р(А)+Р(В)-Р(АВ)
а когда 3 и более событий, вероятность когда они появятся все вместе прибавляется
A+B+C=P(A)+P(B)+P(C)-P(AB)-P(AC)-P(BC)+P(ABC)
Аноним 08/09/17 Птн 12:47:25 #283 №24446 
>>24438
есть множества A, B
как найти их |A∪B|?
|A| + |B| - |A∩B|

если множеств больше чем два:
https://en.wikipedia.org/wiki/Inclusion%E2%80%93exclusion_principle
Аноним 09/09/17 Суб 09:40:15 #284 №24500 
>>24438
Формула включений-исключений. Можно по индукции доказать.
Аноним 09/09/17 Суб 18:15:12 #285 №24517 
>>9338 (OP)
такое уравнение
xlog(5/6)=log(0.3)
смотрю решение, там сразу первая строчка
-x
log(5/6)=-1.2039
почему блэд log(0.3) превратился в -1.2039 когда он равен -0.522
Аноним 09/09/17 Суб 18:22:37 #286 №24518 
>>24517
хотя нет там вот так
-xlog(6/5)=-1.2039
но все равно непонятно почему
Аноним 09/09/17 Суб 18:58:43 #287 №24529 
>>24518
Что такое xlog?
Аноним 09/09/17 Суб 19:00:28 #288 №24530 
>>24518
Скорей всего, под log понимается ln. Но если xlog = x log, то это не важно.
Аноним 09/09/17 Суб 19:05:10 #289 №24532 
>>24517
Без указания основания твой вопрос бессмысленный, однако спешу заметить что Ln(0.3) действительно равен -1.2039..
Аноним 09/09/17 Суб 19:29:11 #290 №24538 
>>24532
Не еби мозг: логарифм определен до свободного действия мультипликативной группы.
Аноним 09/09/17 Суб 19:31:05 #291 №24539 
>>24532
короче говоря, когда ты пишешь равенство ты можешь не конкретизировать изоморфизм
Аноним 09/09/17 Суб 19:54:48 #292 №24542 
>>9338 (OP)
Посоветуйте с чего лучше начать изучение математики?
Аноним 09/09/17 Суб 20:43:00 #293 №24552 
>>24542
Со страпона
Аноним 09/09/17 Суб 21:35:18 #294 №24569 
>>24552
проорал
Аноним 09/09/17 Суб 21:35:43 #295 №24570 
>>24542
С матанализа Зорича
Аноним 09/09/17 Суб 21:36:11 #296 №24571 
>>24542
Или с Лорана-Шварца
Аноним 09/09/17 Суб 21:39:42 #297 №24575 
>>24542
С арифметики, серьезно. Возьми учебник Киселев А.П "Арифметика" дабы по его окончанию смог бы сказать, что уверено умеешь делить десятичные дроби и составлять пропорции на пути к алгебре.
Аноним 09/09/17 Суб 21:57:07 #298 №24582 
>>24575
Спасибо,обязательно ознакомлюсь.Как много в день стоит тратить на математику?Или по своему усмотрению?
Аноним 09/09/17 Суб 21:57:38 #299 №24584 
>>24570
Матанализ?Это о чём?Точнее,чему меня научит это?
Аноним 09/09/17 Суб 22:56:45 #300 №24597 
гетеросексуальный-белый-мужчина.jpg
>>24542
Отрасти патлы, но удали лишнюю растительность с тела, сделай маникюр-педикюр, начни лучше ухаживать за собой в общем плане, купи для начала комплект простых тяношмоток, юбочки-чулочки всякие. Нарядись в него, сделай красивую причёску. Всё, ты готов к началу изучения математики.
Аноним 09/09/17 Суб 22:58:38 #301 №24599 
>>24597
Поясни.
Аноним 09/09/17 Суб 23:18:45 #302 №24604 
>>24584
Сколько тебе лет? Какая база есть?
Аноним 09/09/17 Суб 23:19:35 #303 №24605 
>>24597
проорал х2
Аноним 10/09/17 Вск 00:01:24 #304 №24606 
>>24604
>>24604
18 лет.Что подразумеваешь под базой?
Аноним 10/09/17 Вск 11:43:32 #305 №24609 
>>24606
Очевидно, что речь идёт о твоих текущих математических навыках. Ведь если ты задаешь подобные вопросы, то ты, должно быть, капчуешь из глухой деревни, где интернет провели только месяц назад, либо просто не шаришь от слова вообще, потому что знания и книжки тебе были не интересны, и речь не только о математике. Но уточнить не помешает.
Соответственно, начинать читать тут нужно не аспирантские учебники, а что-нибудь более подходящее для твоей ситуации.
Аноним 10/09/17 Вск 12:47:04 #306 №24613 
>>24609
Ну проблема в том,что у нас в школе учили не думать,а просто решать определённые задачи,чтобы мы на ЕГЭ не опозорили школу,и "база" у нас слабая,вот почему я уточнил.Сам без проблем решаю 1 - 14 задачи ЕГЭ.Хочу научиться решать параметры (нас не учили вообще им),понимать и уметь решать все задания,связанные с графиком функции.
>либо просто не шаришь от слова вообще, потому что знания и книжки тебе были не интересны
По правде говоря,тут ты прав.Начал читать,стремиться получать новые знания этим летом.
Аноним 10/09/17 Вск 17:39:59 #307 №24628 
>>24613
>Хочу научиться решать параметры
Берешь сборник задач Сканави и без задней мысли решаешь.
Аноним 10/09/17 Вск 18:06:26 #308 №24636 
>>24628
Хорошо,можешь посоветовать книгу,для изучения функций,графиков,системы координат?
Аноним 10/09/17 Вск 18:35:06 #309 №24645 
>>24636
Математика: Справочные материалы. Гусев В.А., Мордкович А.Г.
Аноним 10/09/17 Вск 18:41:55 #310 №24651 
>>24645
Спасибо,добра тебе.
Аноним 11/09/17 Пнд 17:40:03 #311 №24690 
Сап аноны. На первом курсе начали дрочить матрицы. Вроде все действия с ними понятны, но вот в чем беда... ЧЕ ЭТО ВООБЩЕ ТАКОЕ? КАК ОНИ ПОЯВИЛИСЬ? КТО ЭТИ ДЕЙСТВИЯ С НИМИ ПРИДУМАЛ? ПОЧЕМУ ВСЕ ФОРМУЛЫ ДАНЫ БОГОМ? НАХУЯ МНЕ ЭТА ТАБЛИЦА ИЗ ЦИФЕРОК?
Что почитать по этой теме, чтобы прошарить и прочувствовать материал, а не в слепую считать эти определители черточками и треугольничками?
Выручайте!
Аноним 11/09/17 Пнд 18:43:46 #312 №24692 
>>24690
https://en.wikipedia.org/wiki/James_Joseph_Sylvester
Читай ссылки в Publications.
Аноним 11/09/17 Пнд 19:14:49 #313 №24693 
>>24690
Системы линейных уравнений.
Аноним 11/09/17 Пнд 21:42:01 #314 №24699 
Спрошу и у вас.
y''+ (a^2)y = tg(x)
При каких а существует единственное непродолжаемое решение с областью определения (-1;3/2).
Я так понимаю, что раз все функции на этом промежутке непрерывны => мы можем поставить сколько угодно задач Коши и каждая будет решаться. И ответ ни при каких. Мне кажется слишком простое решение, чтобы быть правдой. Да и предыдущая задача решалась точно так же, что меня собственно и настораживает. Все выделенное жирным шрифтом - точное задание, возможно я как-то неправильно понимаю область определения?
Аноним 11/09/17 Пнд 22:14:26 #315 №24700 
20170912013946.jpg
Можете расписать доказательство в обе стороны и пояснить каждый шаг? Чисто интуитивно это верный пример, но я не знаю, как правильно расписывать доказательство. Дальше идут примеры еще сложнее, поэтому чтобы доказывать их, мне нужно разобраться с дк-вом примера попроще. Заранее извиняюсь за повернутый на 270 градусов пик.
Аноним 11/09/17 Пнд 23:03:34 #316 №24703 
>>24690
Смотри как это просто. Есть у тебя матрица A (допустим 3х3) и она умножается на вектор x(3х1): Ax. Это значит, что каждая строчка матрицы как вектор умножается на вектор x (скалярное произведение). Получается 3 строчки - 3 произведения - 3 числа. А теперь тебе нужно найти такой вектор х, что эти скалярные произведения равны каким-то числам. При определенных условиях на А (пусть тебя этим на парах дрочат), такая тройка чисел может быть найдена. То есть мы можем решить уравнение (a_1,x) = b_1 и (a_2,x) = b_2 и (a_3,x) = b_3. Смотри что получилось: мы научились делить в пространстве веторов. Теперь смотри дальше. Есть у нас Ах, но х теперь получается как произведение By: х = Ву. Т.о. Ах = А(Ву). И тут вдруг оказывается, что можно сосчитать сначала (АВ), а потом (АВ)у. Именно по тем правилам, которые ты сейчас учишь, анон.
Аноним 11/09/17 Пнд 23:05:14 #317 №24704 
>>24700
Я могу, но я шею сломаю. Выложи нормальную картинку
Аноним 11/09/17 Пнд 23:40:27 #318 №24707 
>>24704
Уже разобрался, спасибо
Аноним 13/09/17 Срд 18:07:26 #319 №24767 
Аноны, помогите! Уже неделю мучаюсь, не могу вспомнить.
:Нужна формула расчета следующей ситуации: Каждый год ты откладываешь по 10к на пенсию под 5% годовых. Откладываешь 40 лет. Все.
Аноним 13/09/17 Срд 22:48:49 #320 №24780 
>>24767
a_n = 1.05 a_{n-1} + 10
Аноним 13/09/17 Срд 23:32:41 #321 №24784 
>>24780
a40=1,05{40-1}+10
Я правильно прочитал?
Аноним 14/09/17 Чтв 02:36:25 #322 №24794 
>>24784
Не, неправильно. Ты получишь всего 1,5 ляма. Через 40 лет это не деньги. Если таки наш верховный главнокоммандующий как и планирует потратит всю нефть на взятие Литвы, то ты за эти деньги и хлеба на неделю не купишь. Нужно учесть инфляцию и индексацию. Вообще на 40 лет никто не строит прогнозы.
Аноним 14/09/17 Чтв 06:11:15 #323 №24796 
>>24794
Эм, к чему ты это все сказал? Мне просто нужна формула расчета таких процентов.
Аноним 14/09/17 Чтв 21:18:52 #324 №24832 
Screenshot2.jpg
Помогите долбоёбу-первокурснику разобраться с дискреткой, пожалуйста. Есть задания (пикрелейтед). Есть конспект с пары, от которого ничем понятным даже не веет. Как начать шарить в дискретке? Что читать? Может кто-то первые 4 объяснить с примером, а-то даже непонятно, как множества эти строить ориентирванные графы
Аноним 14/09/17 Чтв 23:32:36 #325 №24836 
241.png
Сап добрач, выручайте,
задача, разрезать гирю на части и положить на края весов так, что-бы они не начали перевешивать в одну из сторон
Аноним 15/09/17 Птн 00:08:42 #326 №24837 
>>24836
l1 m1 = l2 m2 -- условие равновесия.
m1 + m2 = 38
l1 = 2.5
l2 = 4.5
реши систему.
Аноним 15/09/17 Птн 00:40:16 #327 №24838 
>>24836
>>24837
если 40 и 22 вес блюдц, то вместо первого уравнения:
l1 (m1 + 40) = l1 (m2 + 22)

нужно разрезать на целое число частей?

(m1 + m2)/gcd(m1, m2) = 133 части
gcd, это наибольший общий делитель.

если без веса блюдец, то получается меньше частей.
Аноним 18/09/17 Пнд 20:34:51 #328 №24991 
Screen Shot 2017-09-18 at 8.19.10 PM.png
Пытаюсь вкатиться в матан, правильно ли я решил эти задачи?
1) A + B + C
2) (хз, вроде логично что будет так же как и в первом. но чую что это неправильно)
3) D = (A ∩ B) + (A ∩ C) + (B ∩ C) + (C ∩ A)
Аноним 18/09/17 Пнд 20:35:39 #329 №24992 
>>24991
>3) D = (A ∩ B) + (A ∩ C) + (B ∩ C) + (C ∩ A)
бля, здесь поледняя операция лишняя
Аноним 18/09/17 Пнд 20:52:26 #330 №24993 
>>24836
разделить 18.5 и на 19.5
Аноним 18/09/17 Пнд 22:13:16 #331 №24998 
>>24993
я дебил который забыл что там еще и про условие равновесия
Аноним 18/09/17 Пнд 22:38:41 #332 №25004 
>>24998
Простая система.
{100+2.5x = 99 + 4.5y
{x + y = 38

Ответ:
x=170/7
y = 96/7
Аноним 18/09/17 Пнд 22:39:58 #333 №25005 
>>25004 адресовалось сюда >>24991



Аноним 18/09/17 Пнд 22:40:43 #334 №25006 
>>25005
Сука, блядь.

>>25004 => >>24836
Вот так
Аноним 20/09/17 Срд 01:46:53 #335 №25036 
1495987158070.jpg
Мне нужна помощь с задачами линейного/нелинейного программирования и выпуклыми множествами/функциями (например, нарисовать геометричечкую интерпретацию условий Куна-Такера или решить задачу методом симплекс). Заданий где-то десятка три, и мне нужно немного объяснить как делать первый пример в них и может быть проверить как я справлюсь с остальными. У меня есть картинки милых одетых девочек и может быть что-то еще.
[email protected]
Аноним 20/09/17 Срд 22:50:32 #336 №25070 
>>24991
первое неправильно
Аноним 21/09/17 Чтв 01:03:43 #337 №25072 
>>25070
а что там надо тогда? пересечение всех трех что ли?
Аноним 21/09/17 Чтв 02:06:06 #338 №25073 
>>25072
ну подумой. если элемент принадлежит всем, то он принадлежит и тому,..., и тому. Аналог и - пересечение. Алсо, пиши U вместо +, это трушнее
Аноним 25/09/17 Пнд 01:51:16 #339 №25168 
Ананас, посоветуй какую-нибудь книжку по теории групп и алгебр Ли для аутистов. Чем проще, тем лучше. В идеале, на русском языке и в понятной для физиков терминологии. С меня как обычно
Аноним 25/09/17 Пнд 03:02:23 #340 №25170 
>>25168
Экзамен по стандартной модели в этом семестре чтоли?
Аноним 25/09/17 Пнд 09:02:15 #341 №25176 
>>25168
В.Д. Ляховский, А.А. Болохов. Группы симметрии и элементарные частицы
Аноним 26/09/17 Втр 13:22:04 #342 №25198 
>>25170
>>25170
Да нет, просто хочу вкатиться в эту тему. Выглядит интересно
Аноним 27/09/17 Срд 22:39:15 #343 №25249 
Screenshot3.jpg
Помогите, пожалуйста. В какую сторону воевать? Вообще не идёт в голову ни одно из свойств сочетаний, которое можно было бы пропихнуть для доказательства этого. Что делать? думать
Аноним 29/09/17 Птн 14:14:40 #344 №25303 
>>25249
Такая хуита не доказывается а только выводится.
Аноним 29/09/17 Птн 19:39:19 #345 №25323 
Какая общая формула для количества различных отношений эквивалентности?
Аноним 29/09/17 Птн 19:55:48 #346 №25326 
>>25249
Я не знаю, как это делать, точнее есть идея, но мне лень ее проверять, посчитай c(n+p 2n), а затем придумай подходящую вероятностную задачу и вычисли двумя способами одно и то же число.
Аноним 30/09/17 Суб 21:04:47 #347 №25364 
>>25323
https://ru.wikipedia.org/wiki/%D0%A7%D0%B8%D1%81%D0%BB%D0%B0_%D0%91%D0%B5%D0%BB%D0%BB%D0%B0
Аноним 02/10/17 Пнд 21:16:53 #348 №25430 
Привет, матанусы. Посоветуйте какой-то вариант для численного расчёта формул, набранных в латехе
Аноним 03/10/17 Втр 02:42:27 #349 №25434 
Продублирую вопрос и тут, дабы скорее получить ответ
Сап, матач
Подскажи дурачку: как найти точку на окружности, которая лежит на одной прямой с другой точкой, которая находится не на окружности
Аноним 03/10/17 Втр 03:16:23 #350 №25436 
>>25434
Отбой !
Дурачок додумался
Аноним 08/10/17 Вск 05:58:05 #351 №25645 
Ну и сюда напишу.
Ребят, помогите вкатиться в геометрию 7-8, что читать? Что учить? срок - 14 дней да, я школьник
Аноним 08/10/17 Вск 20:08:22 #352 №25712 
>>9338 (OP)
Вообщем есть 1681 количество клеток, и есть две переменных: x со значением 1.012 и y со значением 1.192. Они распределены равномерно по этому полю. И они жрут друг друга ровно на эту разницу за один ход. Нужно с помощью формулы понять, за какое кооличество времени переменная y займет полностью поел сожрав переменную x. По какой формуле это вычислять? Я помню какие-то прогрессии гемоетрические или арифмитечские есть, но это все не то. Дайте хотя бы наводку, по какой формуле все это вычислить, я в матане вообще ноль, если что.

Вот, для наглядности, о чем я

http://www.netlogoweb.org/launch#http://www.netlogoweb.org/assets/modelslib/Sample%20Models/Biology/Evolution/Altruism.nlogo
Аноним 13/10/17 Птн 09:47:04 #353 №25837 
Сап Антоны
Имеется вопросик по матану про неопределенности(в пределах).
Что это вообще такое? Почему 0/0 - это неопределённость? Ведь мы бесконечно малое число делим на бесконечно малое и, по моей гуманитарной логике, получаю бесконечно малое тобишь 0. Но ведь это не так и от этого просто нужно избавляться, как говорят в вузе. А ещё вот 1 в степени бесконечность? Мне в шкалке говорили, что 1 в ЛЮБОЙ степени - это один. Почему в анализе не так?
Ps. В вузе учат только работать с неопределенностями, но про них ни слова не говорят.
Аноним 14/10/17 Суб 07:31:23 #354 №25848 
>>25837
>Ведь мы бесконечно малое число делим на бесконечно малое и, по моей гуманитарной логике, получаю бесконечно малое тобишь 0.
Рассмотрим предел x/x при x->0, очевидно, что он равен единице, в тоже время твоя "логика" утверждает что это должен быть нуль.
Аноним 15/10/17 Вск 19:31:14 #355 №25893 
IMG20171015192258.jpg
помоги решить
Аноним 15/10/17 Вск 20:20:31 #356 №25900 
>>25837
>бесконечно малое число
В этом твоя проблема. Не бывает "бесконечно малых чисел".

Фраза "бесконечно малое число делим на бесконечно малое" по сути означает "берем две бесконечно малые последовательности/функции (т. е. с пределом, равным 0), и делим одну на другую". В высшей математике 0 на 0 (числа) делить тоже нельзя. Можно делить бесконечно малые последовательности или функции, которые просто для краткости обозначают их пределом, то есть нулем. И правила для такого деления только отчасти совпадают с правилами деления для чисел: аналогия есть, а идентичности нет. И все они доказываются отдельно.

Аналогично с 1 в степени ∞. Имеется в виду последовательность/функция с пределом 1 в степени последовательности/функции с пределом ∞ (т. е. расходящейся в бесконечность). Дальше все как с 0/0: это условность, похожая по многим свойствам на возведение в степень чисел, но им не являющаяся.
Аноним 15/10/17 Вск 20:20:40 #357 №25901 DELETED
Я думаю стать математиком, пишите мне в друганы, помогу решить задачи, постараюсь https://vk.com/kaczynski_theodore
Аноним 15/10/17 Вск 21:16:50 #358 №25905 
1508091355928.jpg
>>25893
Аноним 18/10/17 Срд 17:55:12 #359 №25966 
Без названия45.png
Помогите пожалуйста(немного запутался тк я тупенький :3)
Аноним 18/10/17 Срд 18:02:35 #360 №25967 
>>25966
Шестой класс?
Аноним 18/10/17 Срд 20:29:46 #361 №25972 
KXaD9PGz9xY.jpg
сап, аноны, объясните что делать с синусом в первом задании, с натуральным логарифмом во втором, ну, и если не трудно, то решите 3-е и 4-ое. буду премного благодарен
Аноним 18/10/17 Срд 20:31:28 #362 №25974 
>>25972
|sin| ≤ 1, улавливаешь?
Аноним 18/10/17 Срд 20:33:58 #363 №25975 
>>25974
ты намекаешь на то, что это один из тех случаев, когда можно тупо подставить число (в данном случае 1) вместо x?
Аноним 18/10/17 Срд 20:37:56 #364 №25976 
>>25975
Я намекаю на то, что если f ≤ g, и пределы существуют, то lim f ≤ lim g.
Аноним 18/10/17 Срд 20:40:56 #365 №25977 
>>25976
а можно теперь простым языком?
Аноним 18/10/17 Срд 20:42:51 #366 №25978 
>>25977
Блядь, да, можно заменить синус на 1. Только пока ты не поймешь, почему можно, так и будешь просить решить элементарную хуету на двачах.
Аноним 18/10/17 Срд 20:49:22 #367 №25979 
>>25978
я бы не прочь понять, но мои знания в этой области столь поверхностны, что я не понимаю, где в данном примере эти пресловутые f и g
Аноним 18/10/17 Срд 20:56:55 #368 №25980 
>>25979
Обозначим f = sin, g = 1. f <= g, поэтому lim f <= lim g = 1 (если lim f вообще существует, правда, но в данном случае это не важно). Поэтому, lim sin * h <= lim h для h >= 0. Осталось показать, что lim h = 0 (надеюсь, ты уже догадался, что нужно обозначить за h, а также, как доказать, что ответ еще и >= 0 аналогично).
Аноним 18/10/17 Срд 20:58:24 #369 №25981 
>>25980
И да, расписывать всю эту байду не нужно. Ее надо понять.
Аноним 18/10/17 Срд 21:05:36 #370 №25982 
>>25980
>lim h = 0
если здесь нет опечатки то не понял...
Аноним 18/10/17 Срд 22:21:54 #371 №25984 
Screenshot (7).png
Призываю мамкиных математиков двоща. Хелпаните первокурснику.
Аноним 19/10/17 Чтв 17:20:23 #372 №25997 
>>25984
Не решил еще? Задача скорее на свойства логарифмов, чем на пределы. Вынеси квадрат. Затем верхний логирифм разбей на сумму двух. Потом подели почленно. Одна часть стремится к 0. Вторая - константа 1/ln10. У нас еще остался квадрат, надо в него возвести.
Аноним 19/10/17 Чтв 18:07:09 #373 №26000 
>>25997
Уже решил, но спасибо.
Аноним 19/10/17 Чтв 18:27:13 #374 №26003 
>>25972
Я присоединяюсь. Ну 1 вроде просто, если пользоваться оценкой и эквивалентностью. Второе не смотрел особо. В третьем через эквивалентные убрал логарифм и кое-как перегруппировал знаменатель, но дальше что делать не знаю. В четвертом по асимптотикам раскладываем и упрощаем дробь до вида (1 + a/x), получаем второй зам. предел.
Аноним 22/10/17 Вск 09:56:07 #375 №26091 
Сап, /math/. Как определить количество номеров машин, содержащих две и только две одинаковые цифры (номер машины четырехзначный 0000..9999)?
Аноним 22/10/17 Вск 21:48:48 #376 №26142 
>>26091
ВСЕГО - БЕЗ_ПОВТОРОВ - 3_ОДИНАКОВЫХ - ВСЕ_ОДИНАКОВЫЕ
10^4 - 10 9 8 7 - 4 10 * 9 - 10
Аноним 22/10/17 Вск 21:50:34 #377 №26143 
>>26142
Хотя проще так, наверное:
3 x 2 x 10 x 9 x 8
Аноним 23/10/17 Пнд 08:25:59 #378 №26171 
>>26142
>>26143
Тогда уж ещё 2 пары одинаковых
Аноним 23/10/17 Пнд 10:54:28 #379 №26175 
>>26091
10 (первая цирфра любым способом) x 1 (вторая повторяет первую) x 9 (третья любая кроме первой/второй) x 9 (то же самое) x 4! (перестановки)
Аноним 23/10/17 Пнд 16:49:18 #380 №26180 
Нужны умнейшие! Применение математики в железнодорожном транспорте, дайте статью плеееес
Аноним 23/10/17 Пнд 20:34:36 #381 №26187 
>>26171
Да, проебался в первом варианте.
Аноним 24/10/17 Втр 19:30:33 #382 №26218 
36899168-Math-vector-seamless-texture-with-algebra-plots-tr[...].jpg
>>26187
Привет, математик
Слушайте, хочу начать сечь в математике. Для начала - самой обычной, школьной программе. Дело в том, что я был распиздяем, ничего не учил, сдавал экзамены по алгебре/геометрии на тройки только потому, что сосал хуй преподавателя. Последнее, что помню из школьной программы - это как найти Хуй преподавателя, и то смутно, это какой класс, шестой?
У меня уже полгода постепенно растёт в башке навязчивая мысль начать вкатываться в математику, просто, блядь, неумолимо хочется, и я не знаю, почему. Беспричинно. Нужна помощь советом, анон. Если я начну курить обычные школьные учебники, это будет нормально, чтобы постичь, собственно, школьную программу? Каких авторов учебники предпочесть, кто толковее пишет? Или, может, имеет смысл изучать это дело по каким-то другим методикам? На такие случаи жизни есть ссылки в вашем пастебине?
То есть, для совсем отсталых в этом плане людей, типа меня. Подскажи, анон.
Аноним 24/10/17 Втр 19:48:02 #383 №26219 
>>26180
Ну как минимум планирование расписаний
Аноним 24/10/17 Втр 19:49:20 #384 №26220 
>>26180
https://ru.wikipedia.org/wiki/%D0%A4%D1%80%D0%B0%D0%BD%D1%86%D1%83%D0%B7%D1%81%D0%BA%D0%B0%D1%8F_%D0%B6%D0%B5%D0%BB%D0%B5%D0%B7%D0%BD%D0%BE%D0%B4%D0%BE%D1%80%D0%BE%D0%B6%D0%BD%D0%B0%D1%8F_%D0%BC%D0%B5%D1%82%D1%80%D0%B8%D0%BA%D0%B0
Аноним 25/10/17 Срд 01:10:03 #385 №26237 
>>26218
Да школьные учебники самое то. Они все примерно на одном уровне, скачай несколько, выбери те, которые больше понравятся. Главное делай упражнения, простое чтение тебе ничего не даст. Можешь ещё обратить внимание на khan academy, но это лучше в качестве дополнения.
Аноним 28/10/17 Суб 08:38:00 #386 №26416 
Анон, хочу укрепить свою математическую базу, т.к. не абсолютно не умею доказывать теоремы. Решил начать практически с нуля, класса так с шестого. Стратегия моя весьма проста: дрочить листочки и олимпиадные задачки. Задачки найти не проблема, а вот с листочками беда. Не подскажете где можно достать? Дальше-то НМУ-шные пойдут, а мне бы школьного уровня. Ну или учебники хотя бы для продвинутой школоты, где все строится на определениях и ворохом "докажите что" к ним.
Аноним 28/10/17 Суб 11:55:23 #387 №26419 
>>26416
>>26237
>>26218
Зачем вам учебники, если есть книги типа "алгебры" гельфанда и шеня, гашкова, алексеева?
Аноним 28/10/17 Суб 12:25:34 #388 №26420 
>>26419
Спасибо, анон. Я как раз спрашивал за подобные книги, т.к. не знаю про литературу школьного уровня абсолютно ничего. Если есть еще что близкое к листочкам, то милости прошу.
Аноним 28/10/17 Суб 13:35:42 #389 №26422 
>>26419
Это книги не для нулевых.
Аноним 28/10/17 Суб 18:27:36 #390 №26438 
>>26422
Ну да. Надо уметь складывать в столбик и знать таблицу умножения. Остальное там объясняется, впрочем.
Аноним 28/10/17 Суб 19:21:41 #391 №26445 
>>26438
9/10 с таким набором знаний застрянут на одной из первых задачек в Гельфанде, не говоря уже про остальные две книги.
Аноним 28/10/17 Суб 19:42:18 #392 №26446 
>>26445
>на одной из первых задачек
Типа "умножьте 57 на 10101010101"? Ты эту книгу открывал вообще?
Более того, я тебе скажу что гораздо проще застрять на задаче со звёздочкой в каком-нибудь учебнике Макарычева за седьмой класс. Там кстати дофига задач на доказательство.
Учебников хороших нет вообще, Колмогоров это полная хуйня, у него научишься разве что наивной теории множеств. Лучшее что я видел это Виленкин-Шварцбурд для матклассов, там хоть есть про кольцо многочленов и основную теорему алгебры, но опять же без внятноно определения тех же колец. Учебники это просто кал по умолчанию, лучше не тратить время на них. В крайнем случае есть справочник со всеми школьными фактами и формулами, типа Аленицын-Бутиков, там всего 300 страниц, еще и по физике вся программа, не только математика.
Аноним 29/10/17 Вск 13:56:24 #393 №26485 
>>26416
Специально для маленьких и пушистых https://www.people.vcu.edu/~rhammack/BookOfProof/
Небольшой ликбез про доказательства.
>>26445
Кстати, сдюжил пруф к задаче про ряд Фарея. Не без подсказки, но в целом самостоятельно.

мимо brainlet, который затупил на №44.3
Аноним 29/10/17 Вск 14:32:39 #394 №26487 
>>26485
Интересная задача. Две дроби a/b и c/d для натуральных a, b, c, d соседние, если ad - bc = |1|.
Доказать, что не существует дроби e/f с опять же натуральными e, f, такой что a/b < e/f < c/d; f < b+d.
Всё условие правильно понял?
Пусть a = 2, b = 3, c = 5, d = 7, e = 6, f = 9.
Двп умножить на семь четырнадцать, три на пять пятнадцать, разность 14 - 15 = - 1. Значит дроби соседние.
f = 9 < 3 + 7 = 10, тоже подходит.
Но 6/9 больше чем 2/3 и меньше чем 5/7 (пруф на калькуляторе), что дает контрпример и показывает ложность утверждения.
мимо подумал две минуты
Аноним 29/10/17 Вск 14:41:16 #395 №26488 
>>26487
>6/9 больше чем 2/3
>дает контрпример
Пиздец.
Аноним 29/10/17 Вск 14:44:46 #396 №26489 
>>26488
Двачую. Лучше бы ряды Фарадея посчитал сперва и убедился что множество дробей с натуральными основаниями всюду плотно!
Аноним 29/10/17 Вск 15:45:38 #397 №26491 
>>23077>>22949
Учебник-то есть, конечно, и не один. Английский хорошо знаешь или тебе на русском только?
Аноним 01/11/17 Срд 16:08:09 #398 №26695 
Нужно научиться строить сечения многогранников, где?
Аноним 10/11/17 Птн 23:45:57 #399 №27560 
6 и 5 являются взаимно простыми или же нет?
Аноним 10/11/17 Птн 23:47:37 #400 №27562 
>>27560
Да. Сука, определени загуглить не мог?
Аноним 10/11/17 Птн 23:53:09 #401 №27563 
123.jpg
>>27562
Уточнить зашел. Вкатываюсь потихоньку и блять простейшую хуйню понять не могу.
Но уже вроде разобрался.
Аноним 11/11/17 Суб 00:59:29 #402 №27567 
Как посчитать Binomial[-1,-k] k>0? k целое.
Аноним 11/11/17 Суб 19:17:12 #403 №27627 
>>27567
-1 + k
Бином по определению двухчлен. Два члена "-1" и "k".
Каждый многочлен(тут двухчлен) объединяет одночлены со знаком "+".
Одночлен, может содержать все, кроме операции "+"и "-".
- = +- . Операцию "-" можно заменить "+-". Минус может считаться частью числа.

-1+ k = k - 1, так как +(-1) + k =k +(-1). Коммутативный закон.
a + b = b + a a + (-b) = (-b) + a
1 + 2 = 2 + 1;
1 + (-1) = 1 - 1 = 0
-1 + 1 = 1-1 = 0


Аргумент "k" может принимать любое целое число.
Y = k - 1
Если "k" = 1, то Y = 0, так как Y= k -1 = 1 -1 = 0

Чтобы вычислить полином с целым числом k, нужно от него отнять единицу.
Аноним 14/11/17 Втр 10:14:43 #404 №27756 
Если у нас есть отображение из f: N в N , которое каждому n ставит в соответствие n^2, то почему для такого отображения не существует правого обратного?
Аноним 14/11/17 Втр 12:47:36 #405 №27780 
>>27756
Что такое ''правое'' обратное?
Аноним 14/11/17 Втр 21:19:08 #406 №27810 
photo2017-11-1420-16-47.jpg
Это легкая физика, матач, помоги, пожалуйста.
Аноним 15/11/17 Срд 00:28:57 #407 №27820 DELETED
за сколько реально заботать ЕГЭ по матем на сотку? Оч над, помогите!!!
Аноним 16/11/17 Чтв 05:22:15 #408 №27874 DELETED
>>27810
Иди нахуй, чмо ебаное.
Аноним 17/11/17 Птн 14:08:22 #409 №27969 
BC98F1EE-7DD9-41E9-91C5-9EEC6E4B829D.png
Как это следует-то?
Аноним 17/11/17 Птн 23:26:03 #410 №28045 
>>27560
>>27969
Проиграл с разницы.
Аноним 18/11/17 Суб 20:30:20 #411 №28129 
>>28045
Ну такэто же ОБЗОР, вот и куча примеров где нетрудно увидеть, очевидно, легко понять что.
Помню как Хеллер рыдал от этой книги.
Аноним 19/11/17 Вск 15:16:36 #412 №28192 
56af6736ea0801529d2b2eaf.png
Привет, народ.

Вот пзд нулевый в математике, даже таблицу умножения плохо знаю.
А хочу жутко хотябы бузу знать, но не знаю с чего начать*
Можете дать четкую последовательность изучение великой науки)
Пример:
1.Арифметика
2.Таблица умножения
Аноним 19/11/17 Вск 15:55:14 #413 №28196 
>>28192
Хорошо.
1. Линейная алгебра: Модули, гомоморфизмы, точная последовательность, подмодули и фактор-модули. Свободные модули, базис, размерность. Hom, категория модулей, алгебра эндоморфизмов.
2. Теория Галуа: Тензорное произведение, внешняя степень, определители. Конечные расширения полей как ассоциативные алгебры, полупростые модули, когомологии циклических групп, символ норменного вычета через автоморфизм Фробениуса, теория полей классов, вычисление K(2) поля, плюс-конструкция Квиллена.

Пока хватит. Как освоишь, приходи.
Аноним 19/11/17 Вск 16:12:24 #414 №28197 
>>>28192
Обратись к школьной программе.
Аноним 20/11/17 Пнд 01:08:03 #415 №28226 
>>28192
Книжка "Алгебра" Шеня.
Аноним 20/11/17 Пнд 09:45:31 #416 №28237 
>>28192
Все ищешь в интернете. Учи, все время повторяя.


0.Учишь названия составных частей при добавлении чисел (слагаемое, сумма)
1.Таблица суммы(таблицу плюсования) однозначных чисел
2.Понятие "разряда" числа[десятки, сотни, тысячи]
3. Понятие "класса" числа[тысячи, миллионы, миллиарды и тд]
4.Правила добавления чисел между разрядами.
5. Суммирование в столбик.
ТЕПЕРЬ, ТЫ УМЕЕШЬ ДОБАВЛЯТЬ ЛЮБЫЕ ЧИСЛА ДРУГ К ДРУГУ. ПРАКТИКУЙСЯ. Старайся все делать устно, а не в столбик.

0.Учишь названия составных частей при умножении (множитель, произведение)
1.Таблица умножение(2*3=6)
2.Правило умножение однозначных чисел на двухзначные
3.Правила умножение однозначных на те, которые имеют больший разряд.
4. Правила умножения двухзначного на двухзначное
5.Правило умножение вне зависимости от разрядов.
6.Умножение в столбик
Теперь, ты умеешь умножать. Практикуйся. Старайся без столбика

0.Учишь составные при отрицании. (Уменьшаемое, отнимаемое, разница)
1.Учишь таблицу разницы(знак "-") однозначных чисел
2.Решаешь множество примеров делений, обучаясь таблице деления обратной таблице умножения.
3.Учишь составные при делении(делимое, делитель, частное)

1.Изучаешь понятие кратности на 2,3,6,9,10
2.Изучаешь деление в столбик

1.Изучаешь понятие " отрицательных" чисел(запоминаешь понятие "целых" чисел в отличии от "натуральных")
2.Учишь и практикуешь правила суммирования и отнимании
3.Учишь правило определение знака при умножении

1.Изучаешь основы понятия "дроби", а также термины(числитель, знаменатель, отношение, рациональное число)
2.Изучаешь понятие "простое число"
3.Практикуешься разлагать сложные числа на простые.
4.Учишься находить общий знаменатель.
5.Учишься сокращать дроби
6.Учишься добавлять дроби
7.Учишься умножать дроби
8.Учишься делить дроби
9.Учишься обнимать дроби




Напишешь, когда будешь готов. Ты уже должен будешь хорошо считать целые числа, дроби с любыми операциями.

Дальше можно изучать линейные уравнения, графики функций, квадратные уравнение... Основы геометрии, типа углов, отрезков, прямой, плоскости.

Ищешь, школьные учебники и читаешь параллельно в интернете.



Аноним 20/11/17 Пнд 13:55:09 #417 №28244 
>>28237
>Учишься обнимать дроби
Ня!
Аноним 20/11/17 Пнд 13:58:31 #418 №28245 
>>28237
>9.Учишься обнимать дроби
Это что такое?
Аноним 20/11/17 Пнд 13:59:48 #419 №28246 
>>28237
>Учишься обнимать дроби
Лол.
Аноним 21/11/17 Втр 16:35:08 #420 №28314 
>>13314
Контингент из детишек в основном, в плане насмешек ничего не будет. Я сам поступал в 22, сейчас на втором курсе(МФТИ).
Аноним 21/11/17 Втр 20:41:32 #421 №28325 
>>28314
>Я сам поступал в 22
Для человека, которому тридцать, ты от абитуриента стандартного возраста (семнадцатилетнего школьника) в этот момент не отличался ничем.
Аноним 23/11/17 Чтв 02:19:53 #422 №28445 
EC5FB6D3-3F1A-4FEA-9225-08BC51B3D0E0.png
>>28129
>Ну такэто же ОБЗОР, вот и куча примеров где нетрудно увидеть, очевидно, легко понять что.
Он же пишет не «легко видеть», а «отсюда следует».
>Помню как Хеллер рыдал от этой книги.
Хеллер нуб, поэтому.


Мне кажется, или это вообще очевидно неверно?
Аноним 23/11/17 Чтв 11:45:57 #423 №28454 
>>28445
Легко видеть, что отсюда следует.
Аноним 26/11/17 Вск 21:11:19 #424 №28755 
4ABE72E9-4C5D-4A7D-AE9D-D5344CAD0F87.png
Шестимерное расслаивается над четырёхмерным, слои должны быть двумерными, нет?
Аноним 28/11/17 Втр 01:43:14 #425 №28965 
>>28314
К чему это было сказано? Спрашивал человек 24х лет.
Аноним 28/11/17 Втр 02:21:09 #426 №28966 
>>28965
Пока соберётся, уже 30 стукнет.
Аноним 29/11/17 Срд 10:56:38 #427 №29258 
/sci/ форчана скатился в трэш про мемы и брэйнлетов и притягивает щитпостеров и научпоп как магнит
/sci/ двача аналогично, все хотят понимать кванты по статьям из ленты и википедии
/math/ тут тоже мем на меме, в итоге обсуждать ничего не получается, да и знающих людей удивительно мало
На dxdy/stackexchange дышать свободно нельзя, Physicsforums/reddit получше, но и ими я уже наелся.

В последнее время мне просто становится жалко тратить время на форумы, когда я могу книгу полезную полистать. Кризис среднего возраста? Зачем меня тянет помогать другим на форумах, когда мне никто ни в чём не помогал и я смог всё понять сам?
Аноним 29/11/17 Срд 12:56:20 #428 №29269 
>>29258
>мне никто ни в чём не помогал и я смог всё понять сам
Оттого и тянет, вестимо.
Аноним 29/11/17 Срд 15:29:55 #429 №29280 
>>29258
Дико двачую брейнлетов, пиздец заебали. А вообще, добро пожаловать в постмодернизм.
Аноним 30/11/17 Чтв 03:10:00 #430 №29420 
>>29258
Кстати, мне тут подумалось... а вна фриноде есть соответствующие (живые) каналы?
Аноним 30/11/17 Чтв 22:50:36 #431 №29588 
>>29280
Форчан поначалу кажется забавным и продвинутым, потом от него хочется блевать дальше, чем видишь.
Аноним 01/12/17 Птн 01:17:43 #432 №29602 
>>29280
Годнота, надо здесь зафорсить.
Аноним 03/12/17 Вск 17:12:01 #433 №30190 
15109549761800.png
Волны Де Бройля.jpg
Хелп, плиз.

Волны Де Бройля.
"В соответствии с принятой терминологией говорят, что волны де Бройля связаны с любыми частицами и отражают их волновую природу."

У нас есть неподвижные объект - кристаллическая решётка, в которую жёстко заделаны атомы. Какова будет длина волны Де Бройля для этих атомов? Как искать?
Аноним 04/12/17 Пнд 06:24:09 #434 №30279 
>>29258
а что значит дышать свободно нельзя?
Аноним 04/12/17 Пнд 06:37:35 #435 №30280 
>>30279
Не дают спокойно порассуждать о вечных двигателях и моем видении квантовой механики. Хотя, physics.steckexchange близок к этому.
Аноним 04/12/17 Пнд 11:04:49 #436 №30292 
5.png
Аноны, помогите плис не понимаю почему
данная сумма равна разности сум, верхний предел суммирования которых равны n и m соответсвенно.
Аноним 05/12/17 Втр 13:33:07 #437 №30537 
>>30292
Нарисуй на бумажке ось, на ней отметь 1, n, m (m<n из условия)
Условно обведи части оси, которые соответствуют каждой из трёх сумм
Аноним 06/12/17 Срд 17:23:33 #438 №30648 
Я вот смотрю на "координатный метод", на идею о том, что точки в пространствах можно представлять в алгебраическом виде, порождать всякими уравнениями совокупности точек, которые можно считать геометрическими объектами. Потом заглядываю в манифест Дьедонне. Потом на школьную геометрию. И возникает вопрос: есть ли смысл нубу в геометрии перечитывать школьные учебники? Что-нибудь полезное там есть?
Аноним 06/12/17 Срд 17:37:56 #439 №30653 
>>30648
Ну, формулы площадей, объёмов.
Свойства квадратов, параллелеграммов, трапеций, ромбов, конусов
Свойства n-угольников, окружностей.
Свойства сфер, цилиндров, конусов, пирамид.
Свойства отрезков, плоскостей, пространства.
Свойства препендекуляров, параллельных прямых, точек.
Свойства всяких тетраэдров, октаэдров, гексаэдров, икосаэдров, додекаэдров.
Свойства кубов, параллелепипедов, призм.
Свойства векторов, компларности, коллинеарность, тригонометрическая интерпретация векторов.
Тригонометрические функции, их свойства, их связь с формами и фигурами.

Школьная геометрия харкдорнее, слышал, харкдорнее первого курса вузов.
Вся школьная геометрия, состоит не просто из списка свойств форм пространства, а в сути знания того, как малая часть информации о форме, определяет остальную.
Аноним 06/12/17 Срд 17:45:16 #440 №30654 
>>30648
>И возникает вопрос: есть ли смысл нубу в геометрии перечитывать школьные учебники? Что-нибудь полезное там есть?
Почитай Берже Геометрия, там будет школьная геометрия через алгебру.
Аноним 06/12/17 Срд 18:29:47 #441 №30655 
>>30654
Лютый харкдор.
Эти книги читают Боги?
Аноним 06/12/17 Срд 20:54:35 #442 №30679 
 .png
Что это, blyatt, такое и как это решается?
Аноним 09/12/17 Суб 19:39:22 #443 №31239 
Снимок.PNG
Шалом, в треде для начинающих срет какой-то шизик, так что продублирую тут.
Даун-аутист в треде.
Как найти дифференциал в точке, конечно этой хуйни?
Аноним 09/12/17 Суб 20:19:45 #444 №31249 
>>31239
Не математика
Аноним 09/12/17 Суб 20:32:29 #445 №31252 
>>31249
Ты и тут уже, блядь!
Аноним 09/12/17 Суб 21:01:51 #446 №31257 
>>31239
Что такое дифференциал?
Аноним 09/12/17 Суб 21:41:00 #447 №31278 
>>31257
dx, сорт оф производная, как я понял.
Аноним 09/12/17 Суб 22:20:23 #448 №31285 
>>31257
Оператор в комплексе де Рама. Нильпотентен по умножению со степенью 2.
Аноним 09/12/17 Суб 22:21:27 #449 №31286 
Рассмотрим комплекс:
... —> C(n−1) —> C(n) —> C(n+1) —> ...
где C это дифференциальные формы — то есть элементы алгебры, порожденной символами dx(1), ... dx(n) с соотношениями
(dx)^2 = 0,
dx1 dx2 = - dx1 dx2.
Называется он комплексом де Рама.
Отображение d(n) называется дифференциалом.
Аноним 09/12/17 Суб 22:22:43 #450 №31287 
>>31286
dx1 dx2 = – dx2 dx1
Опечатка. Дифф форма это антисимметрический тензор. Элемент алгебры Грассмана, иначе.
Аноним 12/12/17 Втр 14:12:21 #451 №31484 
Хотел бы понять, как это решается. Первые три еще понятно, где-то сокращенное умножение, где-то дискриминанта, но дальше плохо понимаю. Может кто-нибудь обьяснить, как это решать? Не готовый ответ, а именно как это делается.
Аноним 12/12/17 Втр 14:18:52 #452 №31487 
IMG1812.JPG
>>31484
Аноним 13/12/17 Срд 16:41:43 #453 №31625 
>>31487
(x3-1)/(x-1) тупо делится

sin(4x)/x = 4 sin(4x)/4x = 1 замечательный предел

(корень(1+x)-1)/x
домножить по формуле (a-b)(a+b)=a2-b2
x/(x(корень(1+x)+1)) = 1/(корень(1+x)+1) = 1/2
Аноним 15/12/17 Птн 03:25:17 #454 №31751 
Не сходящаяся последовательность = нефундаментальная последовательность? Ведь любая сходящаяся последовательность фундаментальна.
Аноним 15/12/17 Птн 09:33:40 #455 №31755 
>>31751
Да. Если последовательность фундоментальная, то она сходится.
Аноним 15/12/17 Птн 09:54:54 #456 №31756 
>>31755
>Если последовательность фундоментальная, то она сходится
>Каждая сходящаяся последовательность является фундаментальной, но не каждая фундаментальная последовательность сходится
Аноним 15/12/17 Птн 13:22:46 #457 №31768 
>>31756
Пополнил тебе за щеку и всё сошлось.
Аноним 15/12/17 Птн 15:33:47 #458 №31782 
>>31755
>>31756
Ну, мне препод на паре давал задания, одно из них было написать определение не сходящейся последовательности. Я написал определение нефундаментальной. Это ведь тоже верно?
Аноним 15/12/17 Птн 15:46:32 #459 №31783 
>>31782
В неполных пространствах бывают фундаментальные несходящиеся последовательности. То есть если ты доказал, что последовательность несходящаяся, это ещё не значит, что она нефундаментальная. Определения несходящейся и нефундаментальной поэтому не равносильны.
Аноним 15/12/17 Птн 15:48:44 #460 №31784 
>>31782
То есть спорный случай. Я бы поставил плюс-минус с объяснением, нуль бы ставить не стал. Как поступит твой препод - хз.
Аноним 15/12/17 Птн 16:24:53 #461 №31786 
>>31782
Верно. Если последовательность нефундаментальна, то она не сходится.
Аноним 15/12/17 Птн 16:26:08 #462 №31787 
>>31786
а фундаментальные последовательности не обязаны сходится.
Как пример последовательность приближений sqrt(2) в Q фундаментальна, но не сходится.
Аноним 16/12/17 Суб 10:20:43 #463 №31883 
Расскажите за ваше отношение к компуцкерам?
Вы приверженец высокопроизводительных мощных железок или адепт пера и бумаги ну и вообще в целом, как работаете, какие программы юзаете
Аноним 16/12/17 Суб 17:25:29 #464 №31983 
>>31883
>работаете
>/math
Аноним 20/12/17 Срд 20:28:10 #465 №32471 
>>31787
Как пример последовательность приближений sqrt(2) в Q фундаментальна, но не сходится.

Это с чего бы вдруг она не сходится?
Аноним 20/12/17 Срд 21:50:06 #466 №32481 
>>32471
Корень из двух - иррациональное число, не является элементом Q.
Аноним 21/12/17 Чтв 19:35:08 #467 №32539 
>>32481
Это значит лишь то, что ее предел не принадлежит Q, а не то, что она не сходится.
Аноним 21/12/17 Чтв 19:35:35 #468 №32540 
>>32481
Это значит лишь то, что ее предел не принадлежит Q, а не то, что она не сходится.
Аноним 21/12/17 Чтв 21:34:48 #469 №32559 
>>32539
Любая фундаментальная последовательность точек метрического пространства сходится в пополнении этого пространства. Ты реально не понимаешь?
Аноним 21/12/17 Чтв 22:10:34 #470 №32569 
>>32559
Я понимаю, что последовательнсть приближений корня из двух сходится, а ее предел не лежит в Q, ты реально этого не понимаешь?
Аноним 25/12/17 Пнд 10:27:41 #471 №32938 
image.png
Переотправляю свой вопрос из другого треда для начинающих.
Правда ли, что векторное произведение главных векторов плоскостей задаёт направляющий вектор прямой, по которой эти плоскости пересекаются, при любом базисе пространства? Если нет, то для чего вообще нам может пригодится этот главный вектор в неортогональном базисе?
Аноним 25/12/17 Пнд 22:08:14 #472 №32973 
>>32938
Вроде да.
Аноним 26/12/17 Втр 00:04:01 #473 №32984 
oVCIJgQjIpo.jpg
не знаю, где спросить, поэтому спрошу здесь. у меня немного модифицированная задача о ранце на плоскости

в общем, есть у меня какое-то множество плоских ранцев стандартного размера. и есть какое-то множество прямоугольных (возможно квадратных) плоских вещей, которое мне нужно упаковать в плоские ранцы наиболее эффективно, то есть, таким образом, чтобы занять как можно меньшее число ранцев как можно большим числом вещей. вещи, само собой, не превышают габариты ранца ни по одной из осей

поясню смысл задачи. мне нужно нарезать из листов фанеры нарезать определённое число деталей, из которых я соберу мебель. я при этом хочу сэкономить, то есть, купить как можно меньшее число листов фанеры

какой есть математический инструментарий для решения моей задачи? я бы попробовал применить задачу о ранце, но у меня ранцев несколько
Аноним 12/01/18 Птн 11:39:27 #474 №34452 
/r книжку по комбинаторике, только чтоб совсем для даунов, с красивыми картинками и все такое.
Аноним 12/01/18 Птн 13:35:19 #475 №34459 
>>34452
Виленкин Комбинаторика.
sageТеория чисел  [mailto:sage] Аноним 12/01/18 Птн 20:38:59 #476 №34469 
Посоветуйте плез книгу (учебник) по теории чисел для школьника-старшеклассника / студента .
систематический свёрточный кодер SSk 15/01/18 Пнд 00:17:37 #477 №34856 
image.png
image.png
Сап. Возникли вопросы с ССК. Прошу помочь.
Имеется систематический свёрточный кодер (пик1) R=1/2;
Входные двоичные информационные символы поступают в сдвигающий регистр, состоящий из трех ячеек, находившийся в исходном нулевом состоянии.
После прихода на вход сдвигающего регистра очередного информационного символа коммутатор опрашивает два выхода и формирует тем самым два выходных кодовых символа.
-------
Как я понимаю, все возможные состояния такие:
000 -> 00
001 -> 01
010 -> 00
011 -> 01
100 -> 11
101 -> 10
110 -> 11
111 -> 10
-------
Однако в книге приводится кодовое дерево для данного кодера (пик2) (красным отметил первые расхождения, коих намного больше).
Мне кажется, что приведенное кодовое дерево нарушает тезис: " В случае систематического сверточного кода первым из выходных кодовых символов, получаемых за каждый цикл опроса коммутатора, всегда будет очередной информационный символ, поступивший в сдвигающий регистр."
Может я что-то неправильно понимаю. Хелп
Аноним 15/01/18 Пнд 00:18:47 #478 №34857 
image.png
>>34856
Схема кодера
Аноним 20/01/18 Суб 12:45:41 #479 №35339 
I0gb84GUkcc.jpg
Здарова двач.
Короче у меня ниебецки бомбит пукан из-за говно-шараги в моей мухосрани.
Сегодня был экзамен (термех),там 3 вопроса.
1.1.Теорема о движении центра масс механической системы
Мой ответ: движение механической системы можно представить в виде движения материальной точки(центр масс) в которой сосредоточена вся масса этой системы
1.2.Основной закон динамики
Мой ответ :по второму закону Ньютона - mw=F,и в проекциях расписал .
2.1.Задача(с пика)
расписал через
mw=-cx+F(вынужд)
делил на m и заменял k^2=c/m, F представил через Hsinpt получилось:
-ap^2sin(pt)+k^2sin(pt)=Hsin(pt)
сократил на sin(pt) получил H
дальше X(вын)=H^2/(k^2-p^2)sin(pt)
подставил получил ответ
X(вын)=0.392/(98-49)
sin(7t)
правильно решено?если да то пиздец как у меня бомбит из-за "квалефекации" преподов (а педивикия и учебник Тарга говорят что в вопросах по-крайней мере я прав)


Аноним 20/01/18 Суб 12:46:49 #480 №35340 
>>35339
*H без квадрата в формулы для x
Аноним 22/01/18 Пнд 00:03:12 #481 №35636 
Матемач, мне до усраки нужно решить несколько заданий по методу Стьюдента. Готов оплатить решение анона.
Пишите, кто может, на почту [email protected]
Аноним 27/01/18 Суб 19:59:53 #482 №35958 
PiLXDR1ZmNQ.jpg
kHvIU3uLruE.jpg
двач, меня отчислить хотят. знаю, это моя проблема, но мб найдется анон что поможет мне с данной хуетой.я бы сам разобрался, но я сейчас занят другими предметами, этим займусь в последнюю очередь. заранее спасибо

3 задание на обоих листах
Аноним 28/01/18 Вск 23:34:56 #483 №35996 
Анон, изучаю книжечки из фака уровень школьника. Как эти процессы запаррелить? В школе всё было естественно - мы плыли по течению школьного курса. А теперь надо как-то самому думать. Карочи, есть какие-то гайды по последовательности изучения курса?
Аноним 29/01/18 Пнд 10:30:49 #484 №36055 
Школьник 11 класс, хочу заниматься математикой, с чего начать, если у меня базовые школьные знания(не знаю как оценить объективно свой уровень, но я тот, у кого все списывают в региональной школе, думаю вы меня поняли)
Аноним 01/02/18 Чтв 13:18:08 #485 №36192 
2707309110792657622111154304350125271651711n.jpg
Матемач, поясни мне геометрическую интерпретацию этой формулы
Аноним 01/02/18 Чтв 13:33:05 #486 №36193 
>>36192
проекция непонятной хрени справа на единичный вектор, перпендикулярный двум векторам

почерк - херня полная
Аноним 01/02/18 Чтв 14:31:59 #487 №36196 
>>36193
Это формула расстояния между срещивающимися прямыми
Аноним 01/02/18 Чтв 18:06:23 #488 №36207 
>>35996
Не знаю , что ты там читаешь, но выстроить последовательность можно взглянув на содержание книг. Если не получится, то выручат школьные учебники.
Аноним 02/02/18 Птн 01:33:15 #489 №36226 
>>36207
Хорошо, спасибо. Как думаешь, я добьюсь успеха? :3
Аноним 02/02/18 Птн 16:43:42 #490 №36251 
200px-Ato[1].jpg
>>36226
Аноним 05/02/18 Пнд 21:21:13 #491 №36391 
image.png
2018-02-05-204718.jpg
Как решать подобные задачи (нужно подобрать а, чтобы было одно решение)?
Ответ тут 1, но нашел я его в проге, а не сам.
Аноним 06/02/18 Втр 22:33:37 #492 №36416 
2733269115478168220012567698159868073021537n.jpg
Матемач, помоги мне разобраться как решить эту залупу . В первую очередь не понимаю как решить (0-)^(-3). Что за ебучий 0-?
sage[mailto:sage] Аноним 06/02/18 Втр 22:37:44 #493 №36417 
>>36416
ответ : ноль,вроде очевидно просто..
Аноним 06/02/18 Втр 22:50:43 #494 №36418 
>>36417
А объяснить можешь, пожалуйста?
Аноним 07/02/18 Срд 05:48:23 #495 №36422 
>>36416
>Что за ебучий 0-?
Это надо спрашивать у семинариста/лектора, ибо я бы за такие обозначения ебал бы в сраку. Попробую сванговать: 0- обозначает, что к нулю подходят со стороны отрицательных чисел, т.е слева, аналогично 0+ обозначает, что к нулю подходим со стороны положительных чисел.
Аноним 07/02/18 Срд 07:34:45 #496 №36424 
>>36418
он врёт.
Минус бесконечность.
Видно по графику функции y=1/(x^3).
Это ты в какой стране евросоюза? Ты что там делаешь? Ты что, ебанутый?
Аноним 07/02/18 Срд 10:41:23 #497 №36429 
>>36424
В пшекостане. Да я ебанутый
Аноним 07/02/18 Срд 15:22:55 #498 №36449 
>>36429
Тши пан теж ту мешкайон?

Как там по сравнению с ДС эрафии?
Аноним 10/02/18 Суб 21:31:09 #499 №36569 
>>36449
По сравнению с ДС залупа, но в целом довольно миленько и преподы не уебаны
Аноним 14/02/18 Срд 20:42:54 #500 №36708 
Привет! о/
Начинающий, на досуге от подготовки к экзаменам недавно начал читать аллуфи. Мб у кого-нибудь есть желание как-нибудь порешать задачки из него или откуда-нибудь еще? В принципе можно по любому разделу, но сейчас больше угараю по алгебре.
Аноним 14/02/18 Срд 22:34:48 #501 №36709 
Нужны книги, имеющие следующую структуру: пара-тройка страниц текста - пара-тройка упражнений (желательно с решебником или хотя бы ответами в конце). Из подобного могу назвать курс по исчислению Куранта и курс анализа Шилова. Можно на английском/немецком.
Аноним 14/02/18 Срд 22:44:40 #502 №36710 
>>36709
Напиши сам такую
Аноним 15/02/18 Чтв 15:37:06 #503 №36721 
>>36709
Прасолов Задачи и теоремы линейной алгебры.
http://prasolov.loegria.net/linalg.pdf
Аноним 15/02/18 Чтв 23:10:42 #504 №36739 
>>36708
Каждые пару месяцев стабильно появляется такой желающий почитать аллуфи - и столь же стабильно через недельку пропадает. Попробуй, может у тебя получится лучше остальных.
Аноним 15/02/18 Чтв 23:30:40 #505 №36742 
>>36739
он дико нудный, а ещё фамильярный донельзя, от чего становится ещё нуднее. ему не хватает раскрасок только к тексту, такой там стиль блять

в своё время тоже поддался на рой бесчисленных рекомендаций читать эту книгу

Ван дер Варден - вот классный учебник алгебры, я считаю
Аноним 16/02/18 Птн 09:26:47 #506 №36772 
>>36739
>Попробуй, может у тебя получится лучше остальных.
Тут надо брать какой-то учебник с полным решебником и так проходить, сверяя свои решения. А так почитывать на досуге - просто терять свое время.
Аноним 18/02/18 Вск 16:49:58 #507 №36875 
uoXrBpgAOk.jpg
помогите решить задание по топологии,без него не видать мне зачета
Аноним 18/02/18 Вск 19:54:39 #508 №36877 
1261496199.jpg
Привет завсегдатые. Воощем, учусь на первом курсе ФИЗФАКА МухГУ, естественно преподам поебать, в качестве оттачивания навыков дают по одному номеру каких-нибудь матриц. Не унывая, решил изучать самостоятельно, только вот, столкнулся с проблемой. Я не знаю с чего начать, пробую со Сканави (в школе не решал ничего), идет тяжеловато. Собственно вопрос, как можно отточить навык решения задач, что математических, что физических? На каких задачниках?
Вроде читаю теорию, понимаю, а задачи всё равно не даются.
Извиняюсь, если мой вопрос мог показаться ебанистически тупым для кого-то.
Аноним 18/02/18 Вск 20:53:07 #509 №36882 
>>36877
Берёшь вот это
Аноним 18/02/18 Вск 20:53:28 #510 №36883 
>>36882
Берёшь вот это
http://www.mcnmo.ru/free-books/pdf/alekseev.pdf

А Сканави выкидываешь нахуй
Аноним 18/02/18 Вск 20:53:53 #511 №36884 
>>36877
реально похожи
Аноним 19/02/18 Пнд 00:35:33 #512 №36893 
>>36883
Эту книгу рекомендуют те, кто её не читал.
В ней полезна только первая глава, про группы.
Аноним 19/02/18 Пнд 01:10:56 #513 №36894 
>>9338 (OP)
Тема безнадёжная, но всё же. Если у кого-то вдруг, есть желание побыть сэнпаем, то я бы с великим удовольствием вслушивался в твои премудрости. Сам начиная с дробей хочу дойти до глубоких глубин и готов этому отдать себя полностью, потому что других интересов в жизни у меня не осталось.
Аноним 19/02/18 Пнд 02:42:03 #514 №36895 
>>36894
А ты симпатичная?
Аноним 19/02/18 Пнд 14:49:34 #515 №36903 
>>36895
Чтобы я тогда делала тут, если бы была симпатичной.
Аноним 19/02/18 Пнд 15:13:57 #516 №36904 
>В ней полезна только первая глава, про группы.

Даже это лучше, чем Сканави помноженный Демидович
Аноним 19/02/18 Пнд 16:01:56 #517 №36906 
15190438990190.png
Аноним 19/02/18 Пнд 23:51:09 #518 №36917 
>>36894
Асечку-писечкц
Аноним 20/02/18 Втр 01:03:34 #519 №36919 
>>36904
Никто и не спорит.
Сканави-Демидовичи это для олимпиадоскама. Математики там нет.
Аноним 20/02/18 Втр 10:58:15 #520 №36925 
>>36917
И это математики? Говно какое-то.
Аноним 20/02/18 Втр 13:37:01 #521 №36926 
>>36925
Согласен.
Аноним 22/02/18 Чтв 14:26:15 #522 №37002 
>>36894
Что тебе мешает читать учебник и решать задачи, пидарас?
Аноним 22/02/18 Чтв 14:37:48 #523 №37003 
>>37002
лучший совет и я ему следую Никольский С.М., Потапов М.К., Решетников Н.Н., Шевкин А.В. Алгебра, вечером читаю "Что такое математика" Куранта изо дня в день. Чего же мне тогда не хватает? Общения с человеком который разделял бы мои интересы. Но как и писал выше, тема безнадёжная.

Аноним 22/02/18 Чтв 14:39:15 #524 №37004 
>>37002
Ах да, я не пидорас, ты ошибся.
Аноним 22/02/18 Чтв 15:25:42 #525 №37008 
>>37003
Здесь можешь обсудить всё, что хочешь.
Аноним 23/02/18 Птн 14:00:17 #526 №37024 
>>37003
>разделял бы мои интересы
Это какие интересы?
Аноним 27/02/18 Втр 00:07:04 #527 №37091 
>>37024
Да всё, это ошибка и моя слабость. Извините чуваки, не особо верил в себя. Пока что действительно иду норм, не забиваю и не ленюсь, что для меня удивительно. Даже наращиваю темп усвоения материала. Стали сниться сны с задачками.
Со стороны смешно, но мне за себя впервые приятно. Есть ощущение, что я прикасаюсь к чему-то невероятно охуительному, хотя бы потому, насколько я часто встречаюсь с бесконечностью в примерах, а ведь это только начало!11
А в целом, мне один знакомый кинул мысль, что математика - это язык на котором написана реальность. Для столь потерянного человека как я, подобное изречение стало откровением. И вот, так я попал сюда... потом подумал, что может, есть ещё такие неудачники, вроде меня, но убедился, что вы парни серьёзные.
Аноним 27/02/18 Втр 06:56:15 #528 №37095 
>>37091
>язык на котором написана реальность.
Не надо смешивать реальность и математику. А сны это хорошо, это значит ты впускаешь в себя, проникаешься ими.
Аноним 01/03/18 Чтв 18:14:28 #529 №37173 
image.jpeg
Повторю свой реквест: планирую начать со Сканави, потом Винберг и Зорич.

В след году планирую поступить на второе высшее (теорфиз)

Опытные аноны, оцените
Аноним 02/03/18 Птн 20:39:47 #530 №37223 
image.png
Не знаю какой тред эффективнее, поэтому попытаюсь и здесь.

В общем, я школьник-самоучка, который угорел по математике. И я недавно видел, как можно представить функцию 1/(1-x) в виде суммы 1+x+x^2+x^3... и т.д. Но этот ряд работает только для х по модулю меньше 1. Меня это не устроило, и я "открыл" другой ряд который описывает эту функцию только модуль у числа должен быть больше 1. Ну так вот, мне стало интересно кто и когда до меня уже это открыл. Можете подсказать какой-нибудь сайт или как искать те или иные математические открытия?
Аноним 02/03/18 Птн 21:54:57 #531 №37225 
>>37223
Посмотри ряд Тейлора, по идее вопросы должны отпасть.
Аноним 02/03/18 Птн 22:28:42 #532 №37226 
>>37225
Ряд Тейлора даёт возможность найти ряд который действует в пределах от -1 до 1(не включительно), а я нашёл в пределах (-∞;-1)U(1;+∞). Мне в этом вопросе не совсем интересен ответ (я его и так уже знаю) , меня интересует существование какого-нибудь портала с архивами подобных математический открытий.
Аноним 04/03/18 Вск 17:29:31 #533 №37288 
>>37173
Теорфиз это прикмат? Тогда зачем тебе алгебра? Зорич пойдет, но если окажется сложно (учитывая, что ты начинаешь с "элементарной математики"), можешь попробовать Кудрявцева или вообще книгу по калькулюсу навернуть. Уверен, тебе так же пригодилась бы линейная алгебра, так что замени Винберга на что-то из соответствующего раздела шапки общего треда.
Аноним 04/03/18 Вск 21:32:39 #534 №37295 
>>37226
Чому это ряд Тейлора не дает возможности найти ряд от 1 до inf? Главное чтобы производная была
Аноним 05/03/18 Пнд 21:12:24 #535 №37314 
>>37288
Начал с Дискретной Математики, Хаггарти.

Линал буду изучать после уже, по книге Позняка.

Аноним 16/03/18 Птн 00:38:55 #536 №37582 
Помогите, пожалуйста, придумать монотонную функцию с конечным множеством точек разрыва
Аноним 16/03/18 Птн 19:48:08 #537 №37610 
>>37582
Ты серьезно?
Аноним 23/03/18 Птн 08:54:44 #538 №37856 
Сап, матаны. Посоветуйте хорошую литературу по математическому моделированию, желательно не слишком сложную.
Аноним 26/03/18 Пнд 13:41:04 #539 №37897 
Снимок.PNG
Пасаны, начал повторять матрицы. запнулся на блочных матрицах. Я не совсем понимаю, как выводится правило умножения блочных матриц. Вот смотрите:

http://life-prog.ru/1_36454_blochnie-matritsi.html

в конце есть лихой переход от 1 к 2 пикрилейтед. Объясните доступным языком чисто с формальной точки зрения, как это и почему это верно
Аноним 02/04/18 Пнд 23:15:43 #540 №38089 
15221442083230.jpg
15226969616420.jpg
Сап, добрач. Если бесконечность - это бесконечное множество, которых существует, в свою очередь, бесконечное количество, одно больше другого, то что может быть больше Абсолютной бесконечности, понятие которой ввел Кантор? Больше множеств классы, а что больше классов? И как далеко можно зайти, если следовать по такому индуктивному пути - пути увеличения масштаба? Упремся ли мы во что-то всеобъемлющее, типа Абсолютного Всего/множества всех множеств/метаконцепции/Универсума/Универсальнокласса-всех-классов, включающий в себя любой гипотетически объектневажно, находится ли он в пределах нашего понимания, или нет, но учитывающее все парадоксы, или увеличивать масштаб можно будет бесконечно? В /б/ послали, на фочане тоже, выручайте, братцы, есть и спать не могу, хочу узнать ответ. Спасибо!
Аноним 05/04/18 Чтв 01:21:22 #541 №38149 
>>37173
а первое высшее какое
Аноним 20/04/18 Птн 18:16:51 #542 №38634 
>>38089
Абстракция супермножества смысла не имеет.
Аноним 24/04/18 Втр 22:16:59 #543 №38767 
Пытаюсь учить доказательства. В учебнике задача:

Suppose a is an integer. If 7|4a, then 7|a.


У меня решение такое:

>Proof. Suppose 7|4a. Then 4a = 7x. a = 7x/4; a = 7 * (x / 4). Suppose (x / 4) = 4; Then a = 7d. By definition a is divisible by 7.

Но в учебнике решение этого примера такое:

>Proof. Suppose7|4a.
By definition of divisibility, this means 4a = 7c for some integer c.
Since 4a = 2(2a) it follows that 4a is even, and since 4a = 7c, we know 7c is even. But then c can’t be odd, because that would make 7c odd, not even.
Thus c is even, so c = 2d for some integer d.
Now go back to the equation 4a=7c and plug in c=2d. We get 4a=14d. Dividing both sides by 2 gives 2a = 7d.
Now, since 2a = 7d, it follows that 7d is even, and thus d cannot be odd.
Then d is even, so d = 2e for some integer e.
Plugging d = 2e back into 2a = 7d gives 2a = 14e.
Dividing both sides of 2a = 14e by 2 produces a = 7e.
Finally, the equation a = 7e means that 7 | a, by definition of divisibility.

Верно ли мое решение, и если нет, то почему?
Аноним 24/04/18 Втр 22:17:42 #544 №38768 
>>38767
опечатка
> Suppose (x / 4) = 4
> Suppose (x / 4) = d
Аноним 02/05/18 Срд 01:51:43 #545 №39002 
>>38634
Почему, анон?
Аноним 04/05/18 Птн 21:14:20 #546 №39095 
производная.jpg
Привет, математики. Мне нужно не решение, с ним я и сам справляюсь, а подсказка. Есть функция, надо найти производную. И что-то я безбожно туплю - как относиться в ней к a? Как к константе или переменной? Или я даже не в ту сторону думаю? Сразу скажу, что отучился давно и могу чего-то действительно не помнить, по дружбе помогаю решить пару задачек. В общем, как брать производную конкретно от а?
Аноним 11/05/18 Птн 21:29:30 #547 №39272 
>>39095
Как к константе.
Аноним 14/05/18 Пнд 19:58:58 #548 №39305 
Math.JPG
Сап, математики, буду благодарен если поможете мне с этой задачей, буду благодарен любой помощи:

Компания производит два вида полимерных материалов: полипропилен и полистирол. Для этого она нанимает неквалифицированных, квалифицированных и высококвалифицированных рабочих. Почасовая оплата в у.е. каждого рабочего и количество тонн материалов, производимым каждым рабочим в час, приводятся в таблице.

Рабочие могут производить одновременно оба продукта, например, квалифицированный рабочий может производить 3 тонны полипропилена 3 тонны полистирола каждый час. Поступил заказ произвести за один час 21 тонну полипропилена и 15 тонн полистирола

Требуется определить, сколько рабочих каждой квалификации необходимо нанять, чтобы выполнить заказ с наименьшими затратами на оплату труда. Постройте математическую модель и найдите оптимальное решение.
Аноним 28/05/18 Пнд 21:57:40 #549 №39915 
17315240.jpg
Ребят, учусь на программиста в вузике и матанализ - это самое сложное на 1 курсе. Мало того, что препод пытается нам в головы впихнуть то, что сам выучил на прикладмате за 3 года за год, так ещё и времени нету сидеть подолгу за ним. Изучение матанализа для меня превратилось в сплошную боль, я даже формулу Тейлора банальную с нескольких попыток так и не прошарил по-человечески. Так вот: я уже долго ищу какую-нибудь книгу по матанализу типа "Матанализ для дибилов" где всё объяснено человеческим языком, а не сухими, тяжело перевариваемыми формулировками. Если кому-то из вас известны материалы (очень желательна/ны именно книга/и а не видео) то поделитемь пожалуйста со мной!
Аноним 03/06/18 Вск 15:02:10 #550 №40098 
uoXrBpgAOk.jpg
помогите,осталось 1 задание чтобы сдать топологию
Аноним 05/06/18 Втр 21:16:53 #551 №40197 
>>38149

Юриспруденция.
Аноним 07/06/18 Чтв 18:28:44 #552 №40230 
Безымян232ный.png
Хелпуй ананас. Решаю Сканави потихоньку, и сталкиваюсь с таким примером как на фото. Проблема в том что я не могу понять нужно ли включать число 2 в решение или нет. тк при х=2 ответ ноль, при х принадлежащем отрезку(2;+бесконечность)-ответ корень из икс, и при х принадлежащем отрезку(0;2) - ответ -1корень из х. Ну так писать ли мне что при х = 2 ответ ноль или хватит и того что при х принадлежащем отрезку(2;+бесконечность)-ответ корень из икс, и при х принадлежащем отрезку(0;2) - ответ -1корень из х? в ответах к сканави число 2 не включено.
Аноним 13/06/18 Срд 15:51:46 #553 №40412 
Аноны, с чего начать вкатывание? Сдал на днях последние егэ, думаю теперь заняться нормальной математикой (для себя и чтобы быть готовым к вузу)

Аноним 24/06/18 Вск 13:49:39 #554 №40982 
>>40412
Бамп реквесту и треду.
Посоветуйте, с каких книг закатиться в выш. мат. Пределы, интегралы, диф. уравнения, дискретная математика.
Аноним 03/07/18 Втр 21:29:05 #555 №41202 
Докажите, что касательная к окружности, описанной около
неравнобедренного треугольника, в его вершине пересекает продол-
жение противоположной стороны в середине отрезка, концы которого
совпадают с основаниями биссектрис внутреннего и внешнего углов
при этой вершине. (Не знаю с чего и как начать доказательство, подсказка не особо помогла. Помогите пожалуйста)
Аноним 03/07/18 Втр 21:50:37 #556 №41206 
>>41202
И не подскажите,что нибудь серьезное для прокачивание скилла на доказательства теорем(проблем с использованием матана нет,но доказательства порой тяжко даются)
Аноним 07/07/18 Суб 20:55:51 #557 №41293 
Год мечтаю и пытаюсь понять, но не понял. Речь о репертуарном методе Кнута в Конкретной математике.
В общем, дано рекуррентное соотношение общего вида ф(1)=а, ф(2н)=2(ф(н))+в, ф(2н+1)=2(ф(н))+с.
Автор замечает, что сие суть ф(н)=А(н)а+В(н)в+С(н)с.
Так как это общего вида формула, он начинает подбирать нужные а, в, с, заменять ф(н) на всякие другие в рамках вышеописанных формул. Получал он при этом ... вот тут у меня бомбит хлеще, чем от того, что написал выше, ибо я слоупок. Так что он получал? Систему чего? Вот ах+bу+сz=д я ещё переживу, но у автора стоят различные неизвестные функции от н. Где это читать, учить, понять? В линейной алгебре не увидел похожего. Анонимно, ты ведь умен как Мотидзуки! Помоги!
Аноним 05/08/18 Вск 22:59:49 #558 №41866 
>>41293
Иногда подбором.
Аноним 10/08/18 Птн 08:54:26 #559 №42005 
>>9338 (OP)
Тыкните носом где тут по статистике угорают?
Аноним 15/08/18 Срд 19:36:47 #560 №42160 
>>42005
Вот по прикладной матёше: https://2ch.hk/math/res/19499.html
Отдельный тред по статистике можешь сам создать.
Аноним 16/08/18 Чтв 17:32:52 #561 №42181 
>>26416
Берёшь листочки матшколы и наслаждаешься.
Аноним 18/08/18 Суб 16:26:01 #562 №42226 
Как учить математику если ты тупой и ничего не знаешь?
Аноним 19/08/18 Вск 08:57:17 #563 №42233 
С чего начать изучение математики?
Аноним 19/08/18 Вск 10:04:38 #564 №42234 
>>42226
>>42233
Сидеть и решать задачи, потом проверять себя. Продолжать это вечно.
Аноним 19/08/18 Вск 10:09:16 #565 №42235 
>>42234
Не, я имею в виду, по какой литературе заниматься? Хочется получить основательное понимание и математическую базу, чтобы потом это не мешало изучать физику, например. Знать хотя бы на вузовском уровне.
Аноним 19/08/18 Вск 12:42:14 #566 №42238 
>>42235
Вузовский уровень физики это что? Общефиз или теорфиз? Если первое, то достаточно взять Берклеевский курс и Фейнмановские лекции с задачником к нему. В лекциях там нужная математика разжёвывается.
Аноним 19/08/18 Вск 17:06:29 #567 №42249 
>>42234
Будто можно просто взять и решать задачи. Надо еще знать откуда и какие...
Аноним 21/08/18 Втр 23:52:47 #568 №42326 
А почему бы и не перекатить тред?
Аноним 25/08/18 Суб 17:46:11 #569 №42406 
>>42226
Во время занятий математикой очень важен правильный настрой. Математика любит инициативу, настойчивость и эмоциональную стабильность. А все вот эти мысли - я тупой, ничего не умею, и т.п. - не объективная истина, а, как правило, мнение о вас родителей, училок и прочих биологических отходов, желающих самоутвердиться за ваш счет. С другой стороны, даже профессионалы порой испытывают фрустрацию или раздражение, когда у них не получается какая-то сложная задача. Тогда есть смысл отложить одну задачу и заняться пока другой. Вообще советую погуглить "Правила В.Ю.", это какой-то московский репетитор, составивший список "установок" на продуктивные занятия. Хорошо написал и по делу.
Аноним 13/10/18 Суб 18:31:35 #570 №44050 
Прикрепленный тред на доске направил меня сюда. А в этом треде я расчитывал увидеть FAQ или картиночку с канонической по версии двача математической библиотекой.
Аноним 13/10/18 Суб 23:35:51 #571 №44065 
>>12210
Это украинская математика, там всё можно
Аноним 24/10/18 Срд 23:49:02 #572 №44416 
1016926802.jpg
Что можете сказать про эти два учебника, можно ли с них начинать закатываться человеку, который до этого не имел дело с математикой? Как они по повествованию, не слишком ли сложно для ньюфага будет?

1 - https://www.ozon.ru/context/detail/id/139242570/
2 - https://www.ozon.ru/context/detail/id/139242573/
Аноним 31/10/18 Срд 16:31:45 #573 №44696 
Хотелось бы узнать по поводу начал мат.ана.
С чего обычно начинают и трудно ли это?
Аноним 12/12/18 Срд 01:13:15 #574 №46488 
Назовём дроби a/b и c/d (a, b, c, d – целые положительные числа) «соседними», если их разность (ad – bc)/bd имеет числитель 1, то есть если
ad – bc =1.
Докажите, что в этом случае никакая дробь e/f с натуральными e и f , у которой f<b +d , не находится между a/b и c/d.

Задача из книги Гельфанда. Никак не могу подступиться. Кто подскажет /направит?
Аноним 23/12/18 Вск 18:35:54 #575 №47487 
Screenshot2018-12-23-20-33-49-988com.google.android.apps.do[...].png
Двач, помоги решить второе неравенство
Аноним 18/06/19 Втр 20:06:10 #576 №56180 
>>14446
Это лучшее, что я видел в своей жизни
Аноним 28/06/19 Птн 15:06:11 #577 №56453 
Народ,можете посоветовать учебник по геометрии,желательно с самых азов,что бы все было внятно и новичку вроде меня было легко ее изучать
Аноним 28/08/19 Срд 20:01:07 #578 №58220 
изображение.png
Как искать формулу степени в степени, не нашёл в свойствах степеней
Аноним 07/10/19 Пнд 06:27:59 #579 №59611 
>>58220
На сколько помню там умножение, т.е в кружке 15
Аноним 10/10/19 Чтв 19:54:02 #580 №59815 
>>59611
Да я уже разобралс, пока тут ответа дождалс, на самом деле там надо вычислить 5 в кубе буит 75, это же степень
Аноним 18/10/19 Птн 19:59:28 #581 №60281 
image.png
Нужна помощь ньюфагу.
Понял, что ничего не понял и теперь хочу начать все с самого начала.
Можете посоветовать каких-нибудь убегодных книг для вкатывания в математику с постепенным повышением сложности. Вот прям с самых азов для школьников средних классов.
Важное условие - максимально простой и понятный язык, доступный даже дауну.

В школе учили решать по шаблонам, в универе то же самое. Сидишь, дрочишь дифуры, интегралы, не понимая зачем. Вроде бы даже сдаешь экзамен на 4-5, а через пару месяцев понимаешь, что мало того, что ничего не понял, так теперь еще и забыл. Из всего курса более-менее на начальном уровне понял только тервер, даже порой пригождается в жизни.
Аноним 21/10/19 Пнд 23:59:21 #582 №60430 
>>60281
курсы на степике (там прям по разные есть), mathprofi.ru

https://www.youtube.com/channel/UCz1OkR6uwWeyut2eBU0CG-A

и самое главное найди приминение ей, типо нейроночки или для расчетов в R studio, matlab, весьма стимулирует. Сам жалею шо в колледже забил на матеш, просто толкали и не давали мотивации
Аноним 27/10/19 Вск 12:37:49 #583 №60768 
>>9338 (OP)
Дана функция x(t)=ln(t^2) в пространстве L2 со стандартным скалярным произведением. Для функции x(t) в этом пространстве требуется найти многочлены наилучшего приближения первой и второй степени.
Проблемы возникают на этапе вычисления коэффициентов Фурье. получается, что для первого коэффициента интеграл (1/sqrt(2)*ln(t^2)) в пределах от -1 до 1 равен минус бесконечности. С другим коэффициентами в принципе дела не лучше.
Что делать в таком случае, как дальше подставлять это в формулу для многочленов приближения?
Аноним 01/11/19 Птн 17:35:53 #584 №60988 
Ребят, простая задачка по ТЧ, школьная:
доказать, что сумма значений функции Эйлера на делителях натурального числа равна ему самому. Изи решил "в лоб", воспользовавшись мультипликативностью функции и посчитав для степени простого числа, но просят посчитать комбинаторным способом, а у меня рука на это не набита, хз чё делать, не гуглится( Помогите(
Аноним 08/01/20 Срд 14:30:27 #585 №63714 
IMG20200108142456.jpg
Извиняюсь за то что я такой слоупок,но дорогие аноны,вы не сможете ли подсказать как делать четвертый и пятый номер на этом бланке?С остальными номерами кроме этих двух проблем вроде нет,но здесь прям вообще пиздец какой-то,никаких идей подходящих нет.
Аноним 21/01/20 Втр 15:44:58 #586 №64207 
pl3LrwF55Vo — копия.jpg
5XC4stb2QQY — копия.jpg
>>9338 (OP)
Анонасы выручайте. Вот вам дан пример, решается интеграл по методу приведения трехчлена к канонической формы. В демидовиче ответ сходится, объясняли однокурсники схожим образом, на проверке у препода подобные проверенные задания зачтены. Так вот, почему это верно, ведь у нас dx. Если бы заместо (х-1/4), стояло бы просто х, вопросов нет, используем форму и готово. Но ведь там (х-1/4) и для дальнейшего, нам нужно, например, воспользоваться методом введения под диференциал, т.е. d(x-1/4). Где я обосрался объясните анонасы? И да, почти все онлайн калькуляторы, выдают решение отличное от этого. Поэтому и еще больше сомнений.
Аноним 21/01/20 Втр 15:57:41 #587 №64211 
>>64207
Уже дали ответ, не отвечайте анонасы.
Аноним 22/01/20 Срд 11:03:32 #588 №64254 
Привет. Друзья, такая ситуация. Когда-то давно дропнул техвуз, сейчас работаю программистом. Не так давно открыл для себя НМУ, просто в свободное время смотрел их лекции и решал задачки. В итоге не заметил, как прошёл первый семестр - послушал всё, что было и сделал большую часть задач.
И собственно вопросик вот в чём. У меня же всё ок получается, математика такого сорта (чистая, если хотите) меня всегда привлекала гораздо сильнее, чем любая другая область вообще. Другая область - вообще что угодно: рисование, музыка, спорт, етц.. Ничего прекраснее и увлекательнее лично для себя не нашёл. И вот раз это у меня получается, раз это так нравится, но тем не менее мне уже около 24 годиков, то стоит ли пытаться поступить туда по-нормальному, и будет ли шанс после окончания пойти в аспу какого-нибудь вузика и заняться нормальной научной деятельностью? Или всё оставить так, как есть - прогать (что тоже нравится) и продолжать заниматься в качестве хобби, не делая на учебу особых акцентов?
Аноним 23/02/20 Вск 23:54:01 #589 №65384 
Привет всем математикам.
Мне 19-2 (возможно) лет и мне тяжело смотреть в сторону математики. В школе мне попались больные учителя, которые не могли нормально преподавать предмет, делали это безынтересно и вызывали отвращение. Да и сам я что сейчас что раньше был мягко говоря не сахар. Итак, вопрос. Могу ли "полюбить" матан? Или уже всё запущено возрастом, мозгоёблей?

Почему задался вопросом? Искренне верю что лишь знание математики и логики на должном уровне сможет сделать меня нормальным погромистом (300к/нс), а не очередным педоверстальщиком.
Кстати говоря, сейчас у меня знании математики хуевые плохие. Не даётся техникумом. Я могу заставить себя изучать тему, но от того что я ЗАСТАВЛЯЮ себя учить, становится лишь тяжелее. Может таблеточки посоветуете? Книжки? Собственные стори? Хочу вкатиться просто и не мучаясь понапрасну.
Аноним 29/02/20 Суб 16:42:56 #590 №65671 
>>28237
>Учишься обнимать дроби
И тут я капитулировал!
Аноним 16/03/20 Пнд 09:58:04 #591 №66378 
screenshot.67.png
Поясните престарелому бумеру, почему так можно сделать при доказательстве?
Аноним 04/04/20 Суб 17:31:40 #592 №66961 
>>65384
Либо ты получаешь удовольствие от результата твоего решения, либо нет. Все просто.
А боль в процессе решения - нормально. Мышцы знаешь ли тоже молочной(вроде) кислотой наливаются, когда физуху хуяришь.
Аноним 26/04/20 Вск 23:25:35 #593 №68073 
TdegyFOWQV8.jpg
>>9338 (OP)
Я нулевый в математике, поэтому прошу, не смейтесь. Я не тролль, я просто прошу помощи, именно математику я знаю хуже всего. В общем. Научите как делить в столбик, я не понимаю, пожалуйста.
Аноним 27/04/20 Пнд 02:21:57 #594 №68077 
>>10499
Аноним 13/06/20 Суб 09:22:18 #595 №70039 
Есть ли какой тест, чтобы определить свой уровень знания в математике и алгебре?
Аноним 21/06/20 Вск 20:15:18 #596 №70407 
>>68073
Помню был сайт для подготовки к ЕГЭ для совсем нулёвых. Там были раздельные задания для того, чтобы сдать на 3, 4 и 5 и они разжовывались очень просто. Подскажите, кто знает.
Аноним 26/06/20 Птн 16:29:00 #597 №70595 
хочу вкатится в матан. на матиматику забил класса с 7, сейчас захотелось заняться. Подскажите как вкатится, в каком порядке изучать и где, пожалуйста
Аноним 03/07/20 Птн 14:34:34 #598 №70857 
image.png
image.png
>>9338 (OP)
Сап, математач. Я излазил гугл вдоль и поперёк, но так и не нашёл никаких формул или внятных объяснений тому, как делается то, что требуется в задании с пикрила1. Помогите, умоляю.
d=40, h=50, k(x,y,z)=(30,30,5), a(x,z)=(39,19), b(z)=(19).
Аноним 25/07/20 Суб 13:54:35 #599 №71483 
Сяп, скрытая доска. Существует ли ещё способы показать (не)изоморфизм графов помимо нахождение дополнительных графов?
Да и если порядок вершин не определен, а изначальное отображение не установлено, то как тогда всё это дело доказывается? Читаю Белоусова, у него там только дополнительные графы (судя по всему для очень плотных графов), а так просто индуктивно прослеживается соответсвие вершин и ребёр.
Аноним 29/07/20 Срд 16:19:13 #600 №71682 
Слушайте, я чет не нашел ни одной ссылки на вменяемые каналы. Кто-то может что-то посоветовать? Где достаточно полно освещают школьную программу и на хорошем уровне универскую? Я понимаю, что в ютубе много лекций и если выборочно искать, то найти можно, но хочется чтобы все было в одном месте более менее. Посоветуйте.
Аноним 14/08/20 Птн 23:47:15 #601 №72160 
Сап добропостач. У меня истерика случилась по поводу задач на растворы/смеси/сплавы/движение/проценты и прочее.
Теперь я хочу научиться их решать настолько хорошо, чтобы быть подобно тем людям, которых в школ гнобили и унижали, а теперь они сами кого хочешь унизят (только в плане задач). Поэтому реквестирую задачники, на которых можно натренироваться.

Я шкила, только в 10 класс перешёл. Поэтому если каким-то чудом у вас будут материалы для 7-9 буду искренне благодарен. С меня как всегда.

Аноним 25/08/20 Втр 19:54:58 #602 №72389 
Аноны, какие самые годные русскоязычные учебники по дискретной математике вы знаете и какой минимум нужен для его освоения?
Аноним 17/10/20 Суб 15:30:17 #603 №74906 
>>9338 (OP)
>Привет! У меня высшего мат.образования, поэтому прошу гнилые помидоры не кидать - пишу в треде для начинающих.

Короче, вот такой текст в учебнике:
Замечание (Внимание! Только для особо интересующихся).
Пусть: A — утверждение, «x» — переменная, «ϕ» — выражение.
Возникает впечатление, что справедливо следующее правило:
для построения утверждения Subst(A; x;ϕ) нужно
в утверждении A заменить переменную «x» на выражение «ϕ».
Однако, здесь возникает целый ряд трудностей, связанных с тем, что:
переменная «x» может больше одного раза входить в утверждение A;
переменная «x» в утверждении A может находиться в области действия некоторого квантора по переменной «x»;
переменная «x» в утверждении A может находиться в области действия некоторого квантора по некоторой переменной,
содержащейся в выражении «ϕ».

Subst - подстановка

Я не понмаю, в чем именно состоит ряд трудностей

Последние два пункта - про вхождение в область кванторов - просто если, например, заменить в выражении, в котором x четное число, некоторую переменную на выражение, в котором утверждается, что х- нечетное, (и наоборот) то выражение станет бессмысленным?

Про вхождение более одного раза вообще непонятно
Ну, в Subst(P(x,x,y),x,A)->P(A,A,y)? Разве нет?
Аноним 11/02/21 Чтв 13:09:25 #604 №80181 
>>9338 (OP)
Продублирую


Аноны, направьте, пожалуйста, а то я не могу нагуглить решение:
Для циклического кода даны:
Мощность кода(M) = 125
Количество информационных символов(k) = 16
Вектор ошибки х7
Количество обнаруживаемых ошибок(s) = 1
Количество исправляемых ошибок(r) = 1

Я посчитал:
Кодовое расстояние(d) = 3
Кол-во контрольных разрядов(m) = 5 (нижняя граница по оценке Хэмминга и Варшамова-Гилберта)
Порождающий полином(G(x)) = x5+x4+1
Проверочный полином(H(x)) = x16+x15+x14+x13+x12+x10+x8+x5+x4+1

Проблема:
Как из имеющихся данных найти передаваемое сообщение? По условию его тоже нужно найти. Я не могу понять, как. Это как-то связано с тем, что указанная в условии мощность кода 125 много меньше, чем 2k[=16/sup]?
Аноним 14/02/21 Вск 00:38:09 #605 №80305 
тест
[mаth]5^5[/mаth]
[mаth]sqrt(16)[/mаth]
[mаth]5^(sqrt(16) * 2)[/mаth]
[mаth]log(1024,2)[/mаth]
Аноним 04/04/21 Вск 15:35:24 #606 №81981 
image.png
Анонче, я шкила и я единственный, кто смог в параметры. Так вот, мне прилетает по лицу эта вещь и я окончательно ломаюсь с нее. Да, она олимпиадная, но я хочу в ней разобраться.

Анонче, сможешь ли ты мне подсказать, что я должен разобрать здесь первым?
Аноним 04/04/21 Вск 16:20:39 #607 №81982 
image.png
image.png
Анонче, большие пробелы в знании базы. Какие подводные, если начну с пикрил?
Аноним 27/04/21 Втр 12:23:01 #608 №82888 
Мой максимальный уровень - это тройные интегралы. Поверхностные уже со скрипом проходят. Есть варианты прокачаться на более высокий уровень? Если да, то порекомендуйте норм источники, где на максимально простом уровне расскажут и покажут что почем. До сего момента юзал онли http://mathprofi.ru/ В данынй момент остро стоит проблема с пониманием ТФКП
тригонометрия Аноним 12/05/21 Срд 17:32:13 #609 №83335 
image.png
Как привести это говно к общему знаменателю по человечески? Типо в первой дроби по формулам привидения получается -sinx - 1, а во второй, раскрыв cos2x и подставив вместо единицы осн. триг. тождество получил 2(sinx)^2. ...и чо дальше
Аноним 22/05/21 Суб 16:24:34 #610 №83707 
Приветствую. Мне 17, закончил 10 класс. Хочу понять что такое математика, что она изучает, почему ей занимаются и интересно ли это (да, это субъективно, но я и хочу посмотреть на математику с точки зрения того, кому она интересна). Познания в математике нулевые. Посоветуйте как мне вкатиться в математику ? Именно как в науку, а не чтобы подготовиться к ЕГЭ например, хотя это тоже нужно, но тут в принципе всё понятно что делать и как. Итак, что мне прочитать/посмотреть/послушать, чтобы понять что такое математика и возможно начать её изучение полноценно в будущем ? Заранее большое спасибо.
Аноним 06/10/21 Срд 21:19:34 #611 №87888 
Я в унике пинал хуи на линале и матане, а сейчас что-то проснулось желание наверстать. Какой самый приятный и эффективный способ это сделать? Чьи лекции самые топовые и какие учебники/задачники лучше курить и как? Хочу именно с нуля вникать в своём темпе в каждую теоремку и смысол каждой формулки.
Аноним 07/01/22 Птн 05:03:58 #612 №92052 
>>83335
>
>
(cosx)^2=1-(sinx)^2=(1-sinx)(1+sinx), а в знаменателе у тебя -sinx-1=-(1+sinx), т.е. можно будет сократить на 1+sinx, исключив из корней sinx=-1.
Аноним 17/02/22 Чтв 19:02:53 #613 №94081 
Сап, меня напугала куча терминов в треде по топологии, так что пишу сюда. Суть вопроса: может ли гладкое неориентируемое многообразие без края быть краем какого-то другого многообразия, если да, то всегда ли найдётся такое многообразие? В частности, является ли открытая лента мебиуса краем какого-то многообразия?
Аноним 18/05/22 Срд 09:23:08 #614 №95879 
в столбик.jpg
Как называется эта хуйня?
Аноним 17/06/22 Птн 13:24:11 #615 №96423 
>>64254
У меня такой же вопрос
Аноним 31/07/22 Вск 21:42:11 #616 №97505 
>>9338 (OP)
Я вот задался вопросом. Математика на егэ это же просто детский лепет? Я такой весь из себя крутой готовился 1 (один) год и надеялся на 80+ баллов по итогу 76 и я хоть и рад был что балл для поступления я набрал но было обидно за некоторые задания. Ну так вот как по вашему я просто слаб раз не смог сделать 13 16 17 18 задания или это нормально и математическое будущее у меня есть? Матека мне нравится и я планирую грызть гранит
Аноним 31/07/22 Вск 21:44:40 #617 №97506 
>>95879
то что в уме держишь, а название кому-то нужно?
Аноним 01/08/22 Пнд 01:00:05 #618 №97508 
>>97505
Почему ты не смог их решить? Времени не хватило? Переволновался? Совершил арифметические ошибки? Мало потратил времени на разбор методов решения? Или просто в принципе не понял, как решать, когда задача чуть-чуть отошла от того, что ты нарешивал?
Ответив на эти вопросы, ты наполовину ответишь на свой пост.
Аноним 01/08/22 Пнд 17:05:18 #619 №97538 
>>97508
Я такие вообще никогда не встречал. Выходит я не мог выкрутиться в той ситуации а значит не особо то и смыслю в матеке
Аноним 09/11/22 Срд 00:23:09 #620 №99714 
Хочу попробовать вкатиться в эту вашу математику, но что делать если я даже за начальные классы ничего не знаю/не помню. Есть ли какие видеоуроки для совсем зеленых ньюфагов? Хочу пробежаться хотя бы по 9 классам, а там может и уровень 10- 11 кл, если вкатит это дело
Аноним 24/11/22 Чтв 08:50:46 #621 №99986 
>>99714
Элементарная алгебра, Туманов
Аноним 24/11/22 Чтв 19:00:46 #622 №100013 
V
Аноним 09/01/23 Пнд 21:03:51 #623 №100686 
image.png
Поможете?
Аноним 31/01/23 Втр 15:42:54 #624 №101035 
Доброго времени суток. Пытаюсь решить неравенства а-ля x <= 100log(x) или x > 10^2x, онлайн калькуляторы выдают какие-то омеги. В контексте школьной математики это как-то решается вообще?
Аноним 30/03/23 Чтв 22:29:39 #625 №102072 
Многоуважаемые матаны, спасити памахите! Нужна программа для доказательства тавтологичности формул состоящих из предикатов. Вот это вообще про это?
https://ru.wikipedia.org/wiki/%D0%97%D0%B0%D0%B4%D0%B0%D1%87%D0%B0_%D0%B2%D1%8B%D0%BF%D0%BE%D0%BB%D0%BD%D0%B8%D0%BC%D0%BE%D1%81%D1%82%D0%B8_%D1%84%D0%BE%D1%80%D0%BC%D1%83%D0%BB_%D0%B2_%D1%82%D0%B5%D0%BE%D1%80%D0%B8%D1%8F%D1%85

Поддерживаемые и активно развивающиеся решатели: Alt-Ergo, Barcelogic, Beaver, Boolector, CVC3, DPT, MathSAT, OpenSMT, SatEEn, Spear, STP, UCLID, veriT, Yices, Z3.

Какая из них попроще? Кого еще можно повопрошать на эту тему?
Аноним 26/04/23 Срд 17:12:23 #626 №102433 
image.png
Я вероятно вообще не по теме залетел, но тут умные люди сидят, поэтому...
Можно ли вкатываться в логику с подобной книженции, или есть что-то посодержательнее для ньюфага без каких либо знаний?
Какие знания предварительно потребуются?
Аноним 10/11/23 Птн 23:09:38 #627 №110554 
>>95879
Сначала складывешь 45 и 9, получаешь 54, и складывешь с 30. Короче говоря, ты ещё один десяток держишь в уме, 4 пишешь и один (десяток обычно не говорят слово десяток) в уме
Аноним 10/11/23 Птн 23:10:41 #628 №110555 
>>83707
Сеймы насчёт как науки
Аноним 14/06/24 Птн 19:53:56 #629 №115640 
image.png
Простой вопрос: почему криволинейные/поверхностные интегралы 2 рода записывают в виде P, Q, R .. функций а не допустим какой то одной? Что эти функции обозначают?
comments powered by Disqus

Отзывы и предложения